Land Law SBAQs Flashcards

You may prefer our related Brainscape-certified flashcards:
1
Q

Which one of the following items will usually be considered to be a chattel?

A. Fitted kitchen units

B. Built-in oven and hob

C. Free-standing fridge

D. Bathroom sink

E. Fitted wardrobes

A

Only Option C is correct. See the case of TSB v Botham [1995]

How well did you know this?
1
Not at all
2
3
4
5
Perfectly
2
Q

Which one of the interests listed below is capable of existing as a legal interest in land?

A. An easement

B. An interest of a beneficiary under a trust

C. An estate contract

D. An option

E. A restrictive covenant

A

A is the correct answer

An easement is the only interest in the list that is capable of being a legal interest in land - s.1(2) Law of Property Act 1925 (although certain formalities will have to be complied with for it to be valid).

All the other interests are only capable of existing as equitable interests - s.1(3) Law of Property Act 1925.

How well did you know this?
1
Not at all
2
3
4
5
Perfectly
3
Q

Which one of the following transactions does not require a deed to be valid at law?

A. Grant of a legal lease for two years with immediate possession

B. Sale of an existing legal lease

C. Grant of a legal lease for a term of five years

D. The sale of a freehold

A

The correct answer is A

The grant of a legal lease for two years with immediate possession comes within the exception to s.52 LPA that legal leases for not more than three years can be created orally or in writing - see s.54(2) LPA 1925.

However, the sale of an existing lease is the transfer of a legal estate in land and is required to be by deed.

The grant of a legal lease for a term of five years and the sale of a freehold must be completed by deed.

How well did you know this?
1
Not at all
2
3
4
5
Perfectly
4
Q

Which one or more of the following statements is or are correct?

To obtain credit for this question you must identify all correct statements.

A. All land in England and Wales belongs to the Crown

B. The Crown has the right to occupy all the land in England and Wales

C. There are only two LEGAL estates in land

D. A person owning an estate in land can create out of it a lesser estate

A

The correct answers are- A, C and D

All land in England and Wales does belong to the Crown but the Crown does not have the right to occupy it whilst there is a freeholder or leaseholder in possession.

There are only two legal estates in land, the fee simple absolute in possession (the freehold) and the term of years absolute in possession (the leasehold).

A person owning an estate can create out of it a lesser estate, e.g., the owner of a freehold estate can grant a leasehold estate. “Fee” means that an estate is capable of being inherited and “simple” signifies that it may pass to any class of heir.

How well did you know this?
1
Not at all
2
3
4
5
Perfectly
5
Q

Which one of the following answers is the correct statutory provision to match with the statement below?

All contracts for the sale of an interest in land must be in writing, contain all the agreed terms and be signed by or on behalf of the parties to the contract.

A. s.54(2) Law of Property Act 1925

B. s.2 Law of Property (Miscellaneous Provisions) Act 1989

C. s.52 Law of Property Act 1925

D. s.53(1)(b) Law of Property Act 1925

A

The correct answer is B

s.2 LP(MP)A 1989 governs the formation of contracts for the sale of interests in land

C, s.52 LPA 1925 provides that a legal estate can only be passed by deed;

A, s.54(2) LPA 1925 deals with the creation of legal leases for not more than 3 years

D, s.53(1)(b) deals with the formalities for the express creation of a trust in land.

How well did you know this?
1
Not at all
2
3
4
5
Perfectly
6
Q

Is the following statement True or False?

When a seller vacates the property, he may be able to remove fixtures that were present on the property at the time the contract was entered into.

A

The statement is True

The seller is permitted to remove fixtures present on the property when the contract was entered into, but only if they have reserved the right to remove them in the contract.

How well did you know this?
1
Not at all
2
3
4
5
Perfectly
7
Q

A freehold owner entered an agreement (not in the form of a deed) to grant a five year lease with a tenant. The parties have never formalised the arrangement.

Which of the following answers best describes the circumstances in which equity would intervene to recognise the arrangement?

A. The agreement must state that it is a deed, be signed, witnessed, delivered and the tenant must have clean hands.

B. The agreement must be in writing and signed as the interest is not capable of being legal.

C. The agreement must be in writing, signed, contain all the expressly agreed terms and the tenant must have clean hands.

D. The agreement must be formalised by deed.

E. The agreement must be in writing, signed, contain all the expressly agreed terms and the freehold owner must have clean hands.

A

The correct answer is C

Equity may intervene to recognise the agreement if there is a contract complying with s2 of the Law of Property (Miscellaneous Provisions) Act 1989 (which requires the written contract to incorporate all the expressly agreed terms and be signed by or on behalf of all parties) and the tenant has clean hands (Walsh v Lonsdale).

Option A is not the best answer as it omits the need for the contract to comply with s2 LPMPA, in particular the need for the contract to incorporate all the expressly agreed terms.

Option B is wrong. A lease is capable of being legal.

Option D is not the best answer. A five-year lease requires a deed in order to be legal (s.52 LPA 1925 and s1 LPMPA 1989), but this is not relevant to whether equity will recognise the arrangement.

Option E is wrong as it is the tenant who needs to demonstrate clean hands.

How well did you know this?
1
Not at all
2
3
4
5
Perfectly
8
Q

A client has entered into a contract to purchase a house. After exchange of contracts, the client seeks your advice about whether the seller must leave the built in oven and inset hob both made by the same high end manufacturer as part of an expensive modern kitchen. The items are not mentioned in the contract.

May the seller remove the items before completion?

A. Yes, because the items are likely to be fixtures due to the high degree of annexation.

B. Yes, because the items are likely to be chattels due to the slight degree of annexation.

C. Yes, because the items are the seller’s personal property and always removable.

D. No, because removal of the items would cause the room to be unfit for use as a kitchen and appear to form part of the overall design of the kitchen.

E. No, because any item within a property is permanently part of the land and must not be removed.

A

The correct answer is Option D.

Although the items could be removed without significant damage, it is not known whether they are of standard size or easy to replace.

Once removed, the oven and hob could not be used without similar cabinets to house them and the items appear to form part of the overall design of the kitchen. Hence the purpose of annexation is for the better enjoyment of the land and not the items. The items are therefore fixtures.

Options A and B are not the best answers as they only consider the first element of the two stage test (the degree of annexation).

Options C and E are wrong as neither answer reflects the two stage test.

How well did you know this?
1
Not at all
2
3
4
5
Perfectly
9
Q

Which one or more of the following statements is or are correct?

A. A legal easement created over an unregistered title is automatically binding on subsequent purchasers of that title.

B. When the Bilford Bank Trust Corporation, acting as trustee of the Smith Family trust, sells land with unregistered title belonging to the trust, the purchaser will not be bound by the interests of the Smith Family.

C. An equitable lease created over an unregistered title after 1925 is binding on a purchaser unless he is Equity’s Darling.

D. A purchaser of an unregistered title will not be bound by a pre-1926 restrictive covenant if he is Equity’s Darling

A

Statements A, B and D are correct.

A, Legal rights generally bind the world in the unregistered system.

B, A trust corporation can give a valid receipt for capital money arising on the sale of a trust property (ss 2 & 27 LPA 1925).

C, A post-1925 equitable lease is an estate contract and so should be registered as a C(iv) land charge at the Central Land Charges Department to bind a purchaser.

D, Pre-1926 equitable interests (including covenants and easements) still rely on the doctrine of notice (‘Equity’s Darling’) for their enforceability.

How well did you know this?
1
Not at all
2
3
4
5
Perfectly
9
Q

A freehold owner sells part of their land. Within the transfer (by deed) the buyer promises to use the land for agricultural purposes only.

Which of the following answers best describes the interest that has been created?

A. It is a restrictive covenant which is legal as it has been created by deed.

B. It is a restrictive covenant which is not capable of being a legal interest.

C. It is a legal easement as it has been made by deed.

D. It is a restrictive covenant which is capable of being a legal interest.

E. It is a restrictive covenant and must be created by deed in order to take effect at law.

A

Option B is correct.

A restrictive covenant does not appear in either s1(1) or s1(2) of the Law of Property Act 1925 and is therefore not capable of being legal (s1(3) LPA 1925). The required formality for the creation of a restrictive covenant is in writing and signed (s53 LPA 1925). A deed more than meets this requirement.

Options A, D and E are wrong. Creating a restrictive covenant by deed does not alter the fact that it is equitable by nature.

Option C is wrong. This is a restrictive covenant, ie a promise by one landowner in favour of another landowner that they will not do certain things on the land.

How well did you know this?
1
Not at all
2
3
4
5
Perfectly
9
Q

Which one or more of the following statements is or are correct?

An option to renew a legal lease will be binding on a purchaser of the freehold title if…

A. The freehold has an unregistered title and, before the sale, the option had been registered as a C(iv) land charge at the Central Land Charges Department

B. The freehold has an unregistered title and, before the sale, the option had been registered as a class F land charge at the Central Land Charges Department

C. The freehold has a registered title and, before the sale, the option had been registered as an overriding interest on the title at Land Registry

D. The freehold has a registered title and, before the sale, the option had been protected by registration of a notice in the Charges Register of the title at Land Registry

E. The freehold has a registered title and the tenant with the benefit of the option is in actual occupation of the property at completion of the sale

F. The freehold has an unregistered title and the tenant with the benefit of the option is in actual occupation of the property at the completion of the sale

A

The correct answers are A, D and E

To be binding on a purchaser of an unregistered freehold, an option must be protected by registration as a Class C(iv) land charge at the Central Land Charges Department (Plymouth).

(Class F protects the statutory right of a non legal owning spouse or civil partner to occupy under s 30 Family Law Act 1996).

With a registered title the option will be binding if it is either protected as an interest affecting a registered estate by the registration of a notice in the charges register of the title, or it may be protected as an overriding interest if the tenant is in actual occupation under Schedule 3 paragraph 2 LRA 2002.

The option is an interest in land and that interest becomes overriding if the owner of the interest is in actual occupation of the property. Overriding interests are not entered on the register and the principle does not apply to the unregistered system.

How well did you know this?
1
Not at all
2
3
4
5
Perfectly
10
Q

A freehold owner granted a three year lease to a tenant by deed in return for a one off payment of £30,000. The tenant discovered that the freehold owner’s signature had not been witnessed.

What interest has been created?

A. An equitable lease.

B. A legal lease.

C. A legal easement.

D. A legal parol lease.

E. An equitable easement.

A

Option A is correct.

The facts state that a lease has been created (and so Options C and E are wrong). A lease is capable of being legal (s1(1)(a) Law of Property Act 1925). A deed is normally required to create a legal lease (s52(1) LPA 1925). A deed must meet the requirements of s1 Law of Property (Miscellaneous Provisions) Act 1989. The deed fails this as the freehold owner’s signature had not been witnessed (and so Option B is wrong).

There is an exception the the rule that a deed is required to create a legal lease - the parol lease exception. However this will not apply here as one of the requirements (that there should be no fine or premium) is not met as there is a premium (£30,000 one-off payment) and so Option D is wrong.

The lease will therefore be equitable as it appears to meet the requirements of s2 Law of Property (Miscellaneous Provisions) Act 1989.

How well did you know this?
1
Not at all
2
3
4
5
Perfectly
11
Q

As a result of the Land Charges Act 1925, certain incumbrances must be registered as land charges if they are to bind successors in title. Please match the following incumbrances to their correct land charges class.

  1. Restrictive covenant created on or after 1 January 1926
  2. Equitable lease
  3. Estate contract
  4. A right of occupation under the Family Law Act 1996
A
  1. Restrictive covenant created on or after 1 January 1926
    - Class D(ii)
  2. Equitable lease
    - Class C(iv)
  3. Estate contract
    - Class C(iv)
  4. A right of occupation under the Family Law Act 1996
    - Class F
How well did you know this?
1
Not at all
2
3
4
5
Perfectly
12
Q

Tim holds Blackacre on trust for Bella and Barry. Tim sells Blackacre to Peter.

Which ONE of the following statements is correct?

A. If Blackacre has an unregistered title, Bella’s and Barry’s interest will only be binding on Peter if it was registered as a C(iii) land charge at the Central Land Charges Department.

B. If Blackacre has an unregistered title, Bella’s and Barry’s interest will not be binding on Peter if he is a bona fide purchaser for value of the legal estate without notice of their interest (Equity’s Darling).

C. If Blackacre has a registered title, Bella’s and Barry’s interest will be binding on Peter because equitable interests under a trust are always overriding interests.

D. Whether the title is registered or unregistered, Bella’s and Barry’s interest will not be binding on Peter because he overreached it when he purchased the property.

A

The correct answer is B

Peter has not satisfied the condition for overreaching because he did not pay the money to all the trustees being a minimum of two or, if appropriate, to a trust corporation (see ss 2 and 27 LPA 1925). There was only one trustee, Tim.

With unregistered titles it is not possible to register equitable interests arising under a trust at the Central Land Charges Department. This means that when deciding whether or not the interest is binding on a purchaser in the unregistered system we must ask whether or not the purchaser is Equity’s Darling.

With a registered title, interests under a trust are only overriding interests if the owner of that interest is in actual occupation of the property (Schedule 3 paragraph 2 LRA 2002).

How well did you know this?
1
Not at all
2
3
4
5
Perfectly
13
Q

Select the correct phrase from the list below to complete the following statement:

A seller of a registered title must prove his ownership by providing the buyer with _______________.

A. copies of the title deeds

B. A Land Charges Department search

C. A Land Registry search

D. Official copies of the title from Land Registry

A

The correct answer is D

The seller’s ownership can only be proved by entry of his name on the Proprietorship Register of the title to the land at Land Registry. A seller must provide a buyer with a copy of the title, known as ‘official copies’.

A search of the Land Register will not provide this information.

Title deeds and Land Charges searches are only relevant where the seller owns land with an unregistered title.

How well did you know this?
1
Not at all
2
3
4
5
Perfectly
14
Q

Where the title to the property is registered, a mortgage by deed may either be protected by deposit of the title deeds or by the registration of the charge against the title to the property at Land Registry.

Is this statement True or False?

A

The statement is False -

To be binding on a purchaser of the registered title and to be completed as a legal mortgage, the mortgage must appear as a registered charge in the charges register of the title at Land Registry, as it is a registrable disposition under s 27 LRA 2002

It is not possible to protect a mortgage of a registered title by deposit of title deeds.

How well did you know this?
1
Not at all
2
3
4
5
Perfectly
15
Q

Select the correct authority from the list below to complete the following statement:

Easements created by implication or prescription are enforceable against a purchaser of land with registered title under ________________.

A. s 27 Land Registration Act 2002

B. Sch 3 para 2 Land Registration Act 2002

C. Sch 3 para 3 Land Registration Act 2002

D. Sch 3 para 1 Land Registration Act 2002

E. s 32 LRA 2002

A

The correct answer is C

Such easements are legal but are not registrable as they are not created expressly in a deed. Instead, they are protected as overriding interests under Schedule 3 para 3 Land Registration Act 2002.

Sch 3 para 1 protects legal leases for 7 years or less

Sch 3 para 2 protects interests where the owner of the interest is in actual occupation of the land.

Section 27 refers to registrable dispositions

Section 32 to registration of notices.

How well did you know this?
1
Not at all
2
3
4
5
Perfectly
16
Q

Since 13 October 2003, which one or more of the following legal leases must be registered with their own title (and noted against the superior title) in order to bind a purchaser?
Hide answer choices

A. Lease for 3 years

B. Lease for 10 years

C. Lease for 7 years

D. Lease for 21 years

E. Lease for 22 years

A

The correct answers are B, D and E

Only leases granted for more than seven years must be registered with their own title.

(Under the pre-LRA 2002 rules, the term was twenty one years).

How well did you know this?
1
Not at all
2
3
4
5
Perfectly
17
Q

A buyer recently completed the purchase of an unregistered freehold property (‘the Property’). Yesterday, the buyer moved into the Property and was confronted by the neighbouring landowner who claimed to have a right of way over the Property for 20 years. The neighbouring landowner produced a deed evidencing the right of way. The buyer checked the document and discovered that the signatures had not been witnessed.

Will the buyer be bound by the neighbouring landowner’s right of way?

A. Yes, because the right-of way is a legal right and, therefore, binding on the buyer.

B. No, because the right of way is equitable by nature and, therefore, not binding on the buyer.

C. No, because the right of way is legal but only binding if a Land Charge is registered.

D. Yes, because the right of way is equitable and the buyer is bound by the equitable doctrine of notice.

E. No, because the right of way is equitable and the buyer will only be bound if a Land Charge has been registered.

A

The correct Option is E.

An easement for a fixed duration (20 years) is capable of being legal. A deed is required to create a legal easement. The document does not comply with the formalities for creating a deed because the signatures have not been witnessed. The easement is, therefore, equitable and would need to be protected by registration of a Land Charge.

Option A correctly states the legal principle but does not relate to the facts. The answer is, therefore, wrong.

Option B is partially correct in that it identifies that the easement is equitable but wrong in stating that an equitable easement cannot bind the buyer.

Option C is wrong as a legal easement in unregistered land would bind the world. A Land Charge is only required to protect an equitable easement.

Option D correctly states the legal principle applying to equitable easements created prior to 1926 but the easement in question was created 20 years ago and, therefore, the answer is wrong.

How well did you know this?
1
Not at all
2
3
4
5
Perfectly
18
Q

A woman lives in a house, the registered freehold of which is owned in the sole name of her civil partner. The woman has resided there throughout the 12 year relationship, but she has made no direct or indirect financial contributions towards the house.

The relationship has recently broken down and so she seeks advice from a solicitor. She explains to the solicitor that her civil partner has now moved out of the house and is threatening to sell it. This is concerning her as she would be left homeless if a sale did go ahead.

Which of the following statements best describes the advice which should be given to her by the solicitor in relation to her concerns about a sale of the property?

A. She should immediately register a Class F land charges against the name of the civil partner at the Land Charges Department.

B. She should immediately register a notice in the charges register of the title to the property.

C. She should immediately register a restriction in the proprietorship of the title to the property.

D. So long as she continues to live in the property, a buyer will be bound by her overriding interest as a person in actual occupation, irrespective of whether her interest is registered.

E. A buyer will be able to overreach her interest by paying the purchase money to at least two trustees, irrespective of whether her interest is registered or whether she is living in the property.

A

The correct answer is B

She has home rights under s30 Family Law Act 1996 (FLA) as a non-owning civil partner. As the title to the property is registered, she should protect this interest by registering it as a notice in the charges register. If this is done by the date of registration of a buyer, it will bind the buyer and she cannot be evicted from her home without a court order whilst the civil partnership is still in existence (in practice, it is unlikely that a buyer would proceed with a purchase whilst such a notice remains on the register).

Option A is wrong because the registration of a Class F land charge would only be applicable if the property had an unregistered title.

Option C is wrong because a restriction would only be relevant if she had a trust interest and on the facts it appears that no such trust interest has arisen (and in any event, a restriction simply alerts a buyer to overreach).

Option D is wrong because home rights cannot be protected by actual occupation as an overriding interest under Sched 3 para 2 Land Registration Act 2002 (s31(10)(b) FLA 1996).

Option E is wrong because it is only possible for a buyer to overreach a trust interest, and on the facts no such trust interest has arisen.

How well did you know this?
1
Not at all
2
3
4
5
Perfectly
19
Q

Last month, before emigrating to Australia, the then owner (“the Former Owner”) of a house with unregistered title made an outright gift of the house to his cousin who did not have a home of his own. When the Former Owner bought the house a few years ago, his wife paid the deposit and when doing so she made it clear to her husband, the Former Owner, that in paying the deposit she was not doing so as a gift or a loan. Before accepting the gift of the house, the Former Owner’s cousin was shown round the house by the Former Owner and his wife but the cousin was not aware that the Former Owner’s wife had paid the deposit. The Former Owner’s wife, who did not want to emigrate to Australia, is claiming that she is entitled to an interest in the house in consequence of having paid the deposit.

Is the Former Owner’s wife entitled, as a consequence of having paid the deposit, to an interest in the house that is enforceable against the Former Owner’s cousin who now owns the house?

A. Yes, because the house was gifted to the Former Owner’s cousin.

B. No, because the Former Owner’s cousin was unaware that the Former Owner’s wife had paid the deposit.

C. Yes, because the Former Owner’s cousin was aware that the Former Owner’s wife had been living in the house.

D. Yes, because the house had been the matrimonial home.

E. Yes, because the Former Owner’s cousin should have found out that the Former Owner’s wife had paid the deposit.

A

The correct answer is A

The Former Owner’s cousin, not being a purchaser for value of the legal estate, was bound by the implied trust interest of the Former Owner’s wife arising from her payment of the deposit.

Option B is wrong because it is irrelevant that the Former Owner’s cousin was unaware that the Former Owner’s wife had paid the deposit.

Option C is wrong because it is not relevant that the Former Owner’s wife had been living in the house.

Option D is wrong because it is not relevant that the house had been the matrimonial home.

Option E is wrong because there was no requirement on the part of the Former Owner’s cousin to enquire whether the Former Owner’s wife had paid the deposit.

How well did you know this?
1
Not at all
2
3
4
5
Perfectly
20
Q

A solicitor is acting for the buyer of a freehold property with registered title. The buyer plans to extend the property into the garden area to have a larger kitchen and dining space. The seller has told the buyer that the previous owner of the property purchased it from the person next door, and in the transfer deed promised not to alter the exterior without first getting approval from them.

What will the buyer’s solicitor look for when examining the Land Registry official copy of the title to confirm that there is a binding covenant?

A. A restriction on dealings in the Proprietorship Register.

B. An entry referring to a covenant in the Property Register

C. An entry referring to a covenant in the Charges Register.

D. An entry referring to a restriction on dealings in the Property Register.

E. An entry referring to a covenant in the Proprietorship Register.

A

The correct answer is C

The entry would be a notice referring to the covenant in the Charges Register of Title.

Option A is wrong as the entry should be a notice on the Charges Register of Title not a restriction on dealings.

Option B is wrong as the entry should appear in the Charges Register of Title.

Option D is wrong as the entry should be a notice on the Charges Register of Title and a restriction on dealings would appear in the Proprietorship Register of Title.

Option E is wrong as the entry should appear in the Charges Register of Title

How well did you know this?
1
Not at all
2
3
4
5
Perfectly
21
Q

Which one or more of the following transactions will require registration at Land Registry?

A. A sale of an unregistered freehold title.

B. The grant of a lease, by deed, for a term of 3 years.

C. The transfer of an unregistered freehold estate by means of an assent- (An assent is where PR’s transfer land to a beneficiary under a will)

D. The grant of a lease for a term of 8 years.

A

The correct options are A, C and D

S4 Land Registration Act 2002 contains a list of those transactions which require registration at Land Registry. The list includes the sale, gift, mortgage or assent of a freehold or leasehold which has more than seven years left to run.

How well did you know this?
1
Not at all
2
3
4
5
Perfectly
22
Q

To find out the identity of an owner of a registered title you should look at the Property Register of the title at Land Registry.

Is this statement True or False?

A

The statement is False.

The details of the owner of the property will appear in the Proprietorship Register of the title.

How well did you know this?
1
Not at all
2
3
4
5
Perfectly
23
Q

Which ONE of the following statements is CORRECT?

A. The relevant date for assessing actual occupation for the purposes of Schedule 3 paragraph 2 LRA 2002 is the date of registration of the purchase.

B. The relevant date for assessing actual occupation for the purposes of Schedule 3 paragraph 2 LRA 2002 is the date of completion of the purchase.

C. The relevant date for assessing whether an interest protected by Schedule 3 paragraphs 1 or 3 LRA 2002 is enforceable against a buyer is the date of exchange of contracts.

D. The relevant date for assessing whether an interest protected by Schedule 3 paragraphs 1 or 3 LRA 2002 is enforceable against a buyer is the date of registration of the purchase.

A

The correct answer is B

The relevant date for Schedule 3 paragraph 2 is completion (see Abbey National v Cann). The relevant date for the other overriding interests under paragraphs 1 and 3 is also the date of completion of the purchase.

How well did you know this?
1
Not at all
2
3
4
5
Perfectly
24
Q

Under Schedule 3 paragraph 3 Land Registration Act 2002, a qualifying easement must, since 13.10.06, be within the actual knowledge of the buyer and obvious on a reasonably careful inspection of the land and used within the past year.

Is this statement TRUE or FALSE?

A

The statement is False

The conditions are alternatives, so the easement must EITHER be within the actual knowledge of the buyer OR obvious on a reasonably careful inspection of the land OR used within the past year.

How well did you know this?
1
Not at all
2
3
4
5
Perfectly
25
Q

Which one or more of the following statements are CORRECT?

A. A lease granted by deed for a term of 7 years must be registered as it is a registrable disposition.

B. A right under s 30 Family Law Act 1996 will be protected by Schedule 3 paragraph 2 LRA 2002 if the claimant is in actual occupation.

C. An easement for 30 years created by deed should be registered as it is a registrable disposition but may alternatively be protected by Schedule 3 paragraph 2 LRA 2002 as the dominant owner is in actual occupation.

D. A lease for a term of four years created by the formalities set out in s 2 Law of Property (Miscellaneous Provisions) Act 1989 may be protected by a notice on the register as it is an interest affecting a registered estate (s32 LRA 2002) or alternatively by Schedule 3 paragraph 2 LRA 2002 if the tenant is in actual occupation.

E. A mortgage created by deed must be registered on the charges register of the title it affects to be legal under s 27 LRA 2002.

A

The correct answers are D and E

The lease in statement D is equitable because it does not fall within the parol lease exception, being for more than three years, but is created in accordance with s2 LP(MP)A 1989 - an estate contract. It is an interest affecting a registered estate and is protected by a notice on the charges register. If it is not so protected, it may be elevated to overriding status under Schedule 3 paragraph 2 as it is sufficiently proprietary in nature (and if the other requirements of Schedule 3 paragraph 2 are met).

Statement E is a registrable disposition.

Statement A is wrong as a legal lease for exactly 7 years will be overriding under Schedule 3 paragraph 1 LRA 2002.

Statement B is wrong as a right under s 30 Family Law Act 1996 cannot be protected by Schedule 3 paragraph 2 LRA 2002 - see s 31(10)(b) FLA 1996.

Statement C is wrong as a legal easement for 30 years should be registered as a registrable disposition but cannot be protected by Schedule 3 paragraph 2 LRA 2002 as the dominant owner is not in actual occupation of the land affected.

How well did you know this?
1
Not at all
2
3
4
5
Perfectly
26
Q

Last year the current owners of a farm (with a registered freehold title) signed a deed in which they granted their immediate neighbour, a horse riding school, a right to cross their land. A client bought the farm two months ago and has come to see you because she wants to stop the riding school crossing her land. She says the horses churn up the land and make it very muddy.

What advice would you give the client about whether or not she is bound by the riding school’s easement?

A. The easement is an overriding interest which will bind her under schedule 3 paragraph 1 Land Registration Act 2002.

B. The easement was made by deed, is therefore legal and binds the world, including your client.

C. Your client was not party to the deed creating the easement and is therefore not bound by it.

D. The easement is an overriding interest which will bind her under schedule 3 paragraph 3 Land Registration Act 2002.

E. The easement is a registrable disposition which will only bind her if it was completed by registration at Land Registry.

A

Option E is the correct answer.

The expressly granted easement by deed is a registrable disposition which will only bind your client if it was completed by registration.

Option A is wrong because schedule 3 para 1 relates to short leases.

Option B is wrong because it is applying the rules which govern unregistered land when the farm has a registered title.

Option C is wrong. An easement is a proprietary interest in land which can bind a purchaser of the land even though they were not party to it.

Option D is wrong because schedule 3 para 3 does not apply to expressly granted easements.

How well did you know this?
1
Not at all
2
3
4
5
Perfectly
27
Q

A married couple purchase a registered freehold property with the help of the wife’s parents. The parents contributed half the purchase price and occupy the property. One year after their purchase, the married couple obtain a loan in return for the grant of a mortgage in favour of a lender. The married couple default on the loan and the lender seeks to exercise its power of sale.

Is the lender bound by the interests of the wife’s parents?

A. Yes, the wife’s parents have an overriding interest which is binding on the lender.

B. Yes, the wife’s parents have actual occupation which is binding on the lender.

C. Yes, because the lender has failed to overreach the wife’s parents’ interest.

D. No, because the occupation by the parents occurred after the mortgage was created.

E. No, because the lender has overreached the parents’ beneficial interest under a trust.

A

The correct option is E.

The lender paid capital money to two trustees (the married couple). This had the effect of lifting the beneficial interests of the parents (as a consequence of their contribution to the purchase price) from the property and to the proceeds of sale (ie the loan advance from the lender). Option C is, therefore, wrong.

Option A is wrong because although the parents may have a beneficial interest under a trust protected as an overriding interest, such an interest has been overreached by the lender.

Option B is wrong as occupation alone does not create an overriding interest. This must be coupled with a proprietary interest in land.

Option D is wrong because the facts clearly indicate that the parents were in occupation prior to the advance by the lender.

How well did you know this?
1
Not at all
2
3
4
5
Perfectly
28
Q

A buyer recently completed the purchase of a registered freehold property (‘the Property’). Yesterday, the buyer moved into the Property and was confronted by the neighbouring landowner who claimed to have a lease of the Property for five years from last year and granted by deed.

Will the buyer be bound by the neighbour’s lease?

A. No, because the lease is a legal lease, and so, should have been registered on the charges register.

B. No, because the lease is equitable by nature and, therefore, not binding on the buyer.

C. No, because the lease is equitable and the buyer will only be bound if a notice has been registered.

D. Yes, because the lease is a legal right and, therefore, binds the world.

E. Yes, because it is a legal lease for a term of seven years or less, and so, automatically protected as an overriding interest.

A

The correct option is E.

A lease is capable of being legal. In order to be legal it must be created by deed (s 52(1) LPA 1925). The facts state that the lease is made by deed. Options B and C are, therefore, wrong.

The lease is for five years and, therefore, cannot be registered as a registrable disposition. Option A is, therefore, wrong.

A five-year legal lease is automatically protected as an overriding interest (Sch 3, para 1 LRA 2002). Option D is, therefore, wrong as it relates to unregistered land and the facts state that the Property is registered.

How well did you know this?
1
Not at all
2
3
4
5
Perfectly
29
Q

Paul was the owner of a freehold property called “Homelands”. The property is registered at Land Registry.

In February 2015, Paul made a valid will leaving “Homelands” to his sisters, Tracy and Teresa, to hold on trust for the benefit of his wife for life, remainder to his two adult daughters.

Paul died last month. Tracy and Teresa have asked for advice because Paul’s daughters want the property to be sold but his wife wishes to remain living there and Tracy and Teresa do not know how to deal with the situation.

Which one or more of the following statements is or are CORRECT?

A. Tracy and Teresa can apply to court under s.14 TLATA 1996 to determine whether or not a sale will take place.

B. Paul’s wife can apply to court under s.14 TLATA to determine whether or not a sale will take place.

C. Under s.15 TLATA the court is to have regard to the purpose of the trust.

D. Under s15 TLATA the court is to have regard to the settlor’s intention.

E. Under s.15 TLATA the court is to have regard to the interests of any unsecured creditors of any beneficiary.

F. Under s.15 TLATA the court must act according to the trustees’ wishes if they outnumber the beneficiaries.

A

The correct answers are A, B, C and D

Statement E is wrong because only the wishes of creditors secured on the property are taken into account.

Statement F is wrong because the court does not give any greater weight to the wishes of the trustees than it does to the wishes of the beneficiaries.

How well did you know this?
1
Not at all
2
3
4
5
Perfectly
30
Q

Paul was the owner of a freehold property called “Homelands”. The property is registered at Land Registry.

In February 2015, Paul made a valid will leaving “Homelands” to his sisters, Tracy and Teresa, to hold on trust for the benefit of his wife for life, remainder to his two adult daughters.

Paul died last month. Tracy and Teresa have asked for advice because Paul’s daughters want the property to be sold but his wife wishes to remain living there and Tracy and Teresa do not know how to deal with the situation.

Which one or more of the following statements is or are CORRECT?

A. Either Tracy or Teresa can sell the property and the purchaser of the legal estate will take free of Paul’s wife’s and daughters’ equitable interests.

B. The purchaser must pay the money to all the trustees, being at least two in number or a trust corporation, in order to overreach their interests.

C. The purchaser will be bound by Paul’s wife’s and daughters’ equitable interests if they have entered a restriction on the Register.

D. Paul’s wife could have had an overriding interest which was overreached on a sale by Tracy and Teresa.

A

The correct answers are B and D

purchasers of the legal estate do not automatically take the property free of the equitable interests. However, provided that the purchaser pays the money to all the trustees being at least two in number, in this case to both Tracy and Teresa, he or she will overreach all the beneficial interests even where there is a restriction on the register or where the beneficial interest is an overriding interest.

If overreaching does not occur, a restriction will prevent registration of the buyer as the new proprietor.

How well did you know this?
1
Not at all
2
3
4
5
Perfectly
31
Q

Land is transferred to A and B as “beneficial joint tenants”; A and B provide the purchase price in unequal shares and so equity will imply a tenancy in common.

Is this statement True or False

A

The statement is False.

On the assumption that the four unities are all present, the express declaration will override the equitable presumption- see Goodman v Gallant.

How well did you know this?
1
Not at all
2
3
4
5
Perfectly
32
Q

Jessica, Keith, Lionel and Margaret were friends. In 2015 they purchased the freehold property “Cliff Cottage” for occasional use as a holiday home. The conveyance to them contained an express declaration that they were beneficial joint tenants.

In 2019, Keith wrote letters to each of the others saying that he wanted the property sold so that his share could be paid to him out of the proceeds of sale. His letters were properly delivered. The property was not sold.

Keith died leaving all his property to Nicola.

Which of the following statements correctly describes the effect of Keith’s severance and death on the ownership of the equitable interest?
Hide answer choices

A. Jessica, Lionel and Margaret now hold the entire equitable interest as joint tenants.

B. Nicola becomes a beneficial tenant in common as to one-quarter and the remaining three-quarters are held by Jessica, Lionel and Margaret as joint tenants.

C. Nicola becomes a beneficial tenant in common as to one-third and the remaining two-thirds are held by Jessica, Lionel and Margaret as joint tenants.

D. Nicola, Jessica, Lionel and Margaret now hold one-quarter each as tenants in common.

A

The correct answer is B

Only Keith becomes a beneficial tenant in common (as to one-quarter) when he serves the notice, and the remaining three-quarters are held by the other three original parties as joint tenants. When Keith dies, his property passes to Nicola, so she becomes a tenant in common in his place.

How well did you know this?
1
Not at all
2
3
4
5
Perfectly
33
Q

Jessica, Keith, Lionel and Margaret were friends. In 2015 they purchased the freehold property “Cliff Cottage” for occasional use as a holiday home. The conveyance to them contained an express declaration that they were beneficial joint tenants.

In 2019, Keith wrote letters to each of the others saying that he wanted the property sold so that his share could be paid to him out of the proceeds of sale. His letters were properly delivered. The property was not sold.

Keith died leaving all his property to Nicola.

In 2021 Jessica, by a Deed of Gift, gave her beneficial interest to Nicola. In 2022 Lionel died leaving all his property to Nicola.

Is the following statement True or False?

Both Jessica’s and Lionel’s gifts will pass their beneficial shares in the property to Nicola.

A

The statement is False.

Beneficial interests held on a joint tenancy cannot pass by will. So, Jessica’s lifetime gift will be effective as an example of severance by alienation which complies with s53(1)(c) LPA 1925, but Lionel’s gift in his will does not pass his share to Nicola.

How well did you know this?
1
Not at all
2
3
4
5
Perfectly
34
Q

Jessica, Keith, Lionel and Margaret were friends. In 2015 they purchased the freehold property “Cliff Cottage” for occasional use as a holiday home. The conveyance to them contained an express declaration that they were beneficial joint tenants.

In 2019, Keith wrote letters to each of the others saying that he wanted the property sold so that his share could be paid to him out of the proceeds of sale. His letters were properly delivered. The property was not sold.

Keith died leaving all his property to Nicola.

In 2021 Jessica, by a Deed of Gift, gave her beneficial interest to Nicola. In 2022 Lionel died leaving all his property to Nicola.

Which one of the following statements is CORRECT?

A. If a sale of the property takes place, Margaret will be entitled to a quarter of the proceeds of sale and Nicola will be entitled to the remaining three quarters.

B. If a sale of the property takes place, Margaret will be entitled to three quarters of the proceeds of sale and Nicola will be entitled to the remaining quarter.

C. If a sale of the property takes place, Margaret and Nicola will each be entitled to receive half of the proceeds of sale.

A

Statement C is correct.

Nicola took Keith’s share after he severed his beneficial interest and she gets Jessica’s share under her life-time gift. Margaret already had her own interest in the property and she acquired Lionel’s when he died by right of survivorship.

How well did you know this?
1
Not at all
2
3
4
5
Perfectly
35
Q

Jessica, Keith, Lionel and Margaret were friends. In 2015 they purchased the freehold property “Cliff Cottage” for occasional use as a holiday home. The conveyance to them contained an express declaration that they were beneficial joint tenants.

In 2019, Keith wrote letters to each of the others saying that he wanted the property sold so that his share could be paid to him out of the proceeds of sale. His letters were properly delivered. The property was not sold.

Keith died leaving all his property to Nicola.

In 2021 Jessica, by a Deed of Gift, gave her beneficial interest to Nicola. In 2022 Lionel died leaving all his property to Nicola.

Jessica has now died. Nicola is in occupation of the property.

Assuming the title to the land is unregistered, which one of the following statements is CORRECT?

A. Nicola and Margaret hold the legal estate as joint tenants on trust for themselves as tenants in common in equity. If the land is sold by them the sale will overreach the equitable interests arising under the trust as the sale will be effected by all of the trustees being at least two in number.

B. Margaret holds the legal estate on trust for herself and Nicola as tenants in common. If Margaret sells the land a purchaser will take subject to Nicola’s equitable interest because Nicola has gone into occupation and so has an overriding interest under Schedule 3 paragraph 2 LRA 2002. It is not possible to overreach an overriding interest.

C. Margaret owns the legal estate for herself and Nicola as tenants in common in equity. If she sells the land, a purchaser risks buying subject to Nicola’s equitable interest (the purchaser may have constructive notice of Nicola’s equitable interest as she occupies the house). Margaret should first appoint a second trustee to overreach Nicola’s equitable interest.

A

statement C is correct

The only way for Margaret to sell is for her to appoint a second trustee. Statement A is false as Nicola is not on the legal title; she has an equitable interest only. Statement B is incorrect as the title is unregistered so the LRA 2002 will not apply (if this were registered title, note that it is possible to overreach an overriding interest under Schedule 3 para 2 LRA 2002 if it is protecting an interest under a trust).

How well did you know this?
1
Not at all
2
3
4
5
Perfectly
36
Q

Two brothers bought a property together and both their names were entered on the proprietorship register. The only entries on the proprietorship register are (1) their details and (2) the purchase price paid for the property. One of the brothers has died (‘the deceased’) and the other brother is selling the property to your client.

Which one of the following statements represents the best advice to give your client about what steps must be taken to enable your client to register his title at the Land Registry?

A. It will be necessary to overreach the deceased’s beneficial interest as the beneficial title was held by both brothers as tenants in common.

B. It will not be necessary to overreach the deceased’s beneficial interest as the beneficial title was held by both brothers as joint tenants.

c. It will not be necessary to overreach the deceased’s beneficial interest as the beneficial title was held by both brothers as tenants in common.

D. It will be necessary to overreach the deceased’s beneficial interest as the beneficial title was held by both brothers as tenants in common and it will be necessary to appoint a second trustee to enable a valid receipt to be given to your client.

E. It will not be necessary to overreach the deceased’s beneficial interest as the beneficial title was held by both brothers as joint tenants but evidence of the deceased death will be required when your client’s title is registered at the Land Registry.

A

The correct statement is Statement E.

In the absence of a restriction on the proprietorship register in a co-ownership situation the beneficial interest is held as joint tenants and the rule of survivorship applies. Options B and E are both correct but Option E is the best answer as evidence of death will be needed to prove who is the sole surviving legal and beneficial owner. Options A and D are therefore wrong.

Option C is also wrong as even if it were held as a tenant in common the deceased’s beneficial interest would need to be overreached.

How well did you know this?
1
Not at all
2
3
4
5
Perfectly
37
Q

A solicitor acts for a man and his partner in the purchase of a freehold property (‘the Property’) for £350,000. The partner contributed £150,000 and the man contributed the remaining £200,000. The Property is transferred to them as express beneficial joint tenants in equity.

Which of the following best describes how the Property is held?

A. The Property is held legally and beneficially as joint tenants due to the express declaration of trust.

B. The Property is held as a tenancy in common due to the unequal contributions made to the purchase price.

C. The equitable interests are held as a joint tenancy as the express declaration of trust is conclusive.

D. The equitable interests are held in a tenancy in common as the presumption for a joint tenancy has been rebutted by unequal contributions.

E. The Property is held as a joint tenancy as the four unities are present.

A

The correct answer is Option A.

The man and his partner hold the legal estate as trustees as joint tenants (s 1(6) LPA 1925). The four unities are present which would indicate a joint tenancy (or a tenancy in common).

Option E is correct but is not the best answer as it does not reflect the express declaration.

There is an express declaration that the man and his partner hold the equitable interests as joint tenants. Option C is, therefore, correct, but not the best answer as it does not take account of how the legal estate is held.

The express declaration is conclusive, notwithstanding the unequal contributions made to the purchase price. Options B and D are, therefore, wrong.

How well did you know this?
1
Not at all
2
3
4
5
Perfectly
38
Q

Three postgraduate students (a biologist, a chemist and an engineer) bought a house to live in together three years ago. They held the legal estate and the beneficial interests as joint tenants.

Six months ago the biologist decided that she wanted to move out and sold her interest to the chemist, complying with the relevant statutory formalities. Unfortunately the engineer was killed in a car crash last week. Her will left all of her estate to her sister.

Which of the following options best describes how the house is now held?

A. When the biologist sold her interest to the chemist, this severed the legal estate and the beneficial interests. The chemist is now the sole legal and beneficial owner.

B. When the biologist sold her interest to the chemist, this had no effect on the legal estate. The beneficial joint tenancy was severed leaving the chemist and the engineer as joint tenants (2/3) and the chemist as tenant in common (1/3). When the engineer died this left the biologist and the chemist as joint tenants of the legal estate and the chemist as the sole beneficial owner.

C. When the biologist sold her interest to the chemist, this had no effect on the legal estate. The beneficial joint tenancy was severed leaving the chemist and the engineer as joint tenants (2/3) and the chemist as tenant in common (1/3). When the engineer died, her legal interest passed by survivorship to her sister and her beneficial interest passed by will to her sister. The legal owners are the chemist and the sister as joint tenants and the beneficial interests are held by the chemist (2/3) and the sister (1/3) as tenants in common.

D. When the biologist sold her interest to the chemist, this had no effect on the legal estate. The beneficial joint tenancy was severed leaving the chemist and the engineer as joint tenants (2/3) and the chemist as tenant in common (1/3). When the engineer died, this left the biologist, the chemist and the sister as joint tenants of the legal estate and the chemist (2/3) and the sister (1/3) as tenants in common in equity.

E. When the biologist sold her interest to the chemist, this had no effect on the legal estate. The beneficial joint tenancy was severed leaving the chemist (2/3) and the engineer (1/3) as tenants in common. When the engineer died, this left the biologist and the chemist as joint tenants of the legal estate and the chemist (2/3) and the sister (1/3) as tenants in common in equity.

A

Option B is correct.

When the biologist sold her interest in the house to the chemist six months ago this had no effect on the legal estate, which cannot be severed (s1(6) LPA 1925). When the engineer died last week the legal estate passed by survivorship to the biologist and the chemist, who are now the trustees.

The beneficial joint tenancy was severed by alienation because the formalities for a disposition of an equitable interest in s.53(1)(c) LPA 1925 were satisfied when the biologist sold her share to the chemist six months ago. This left the chemist and the engineer as joint tenants of 2/3rds and the chemist as tenant in common of 1/3rd. When the engineer died last week her interest passed by survivorship to the chemist (her will was of no effect in relation to the house), leaving the chemist as the sole beneficial owner.

How well did you know this?
1
Not at all
2
3
4
5
Perfectly
39
Q

A wife and husband bought their family home 20 years ago. The title was registered and, because they were beneficial joint tenants, there was no restriction in the proprietorship register. The husband died last month and the wife is now selling the house. The husband’s will left all of his estate to the couple’s daughter. The buyer is considering what he needs from the seller’s solicitor in relation to the husband’s death.

Which of the following statements best describes what the buyer needs?

A. He will need to see the husband’s death certificate and will.

B. He will need to see the husband’s death certificate, will and he will require that the daughter is party to the sale.

C. He will need to require that the wife appoints a second trustee to join in the sale in order to overreach the daughter’s beneficial interest.

D. He will need to see the husband’s death certificate.

E. He will need to see the husband’s death certificate, will and he will require that the husband’s personal representatives join in the sale.

A

Option D is correct.

The wife and husband were joint tenants of both the legal estate and the equitable interest so, on the husband’s death, both passed by survivorship and the wife became the absolute owner of the house. All that you will need to see is the husband’s death certificate.

40
Q

Which ONE of the following statements is Correct?

A. Positive covenants may be enforced against successors in title to the original covenantor.

B. Restrictive covenants can be enforced against successors in title to the original covenantor at both common law and in equity provided that various conditions are satisfied.

C. Both positive and restrictive covenants can be indirectly enforced against successors if a chain of indemnity exists.

A

C is correct.

Neither positive nor restrictive covenants can be enforced against successors in title to the original covenantor at common law.

Restrictive covenants can only be enforced in equity under the rule in Tulk v Moxhay.

41
Q

Which ONE of the following statements is Wrong?

A. The annexation of the benefit of a covenant attaches the benefit of the covenant to the land of the covenantee

B. The annexation of the benefit of a covenant occurs at the same time as the transfer of the land to a successor covenantee.

C. The annexation of the benefit of a covenant occurs at the time of creation of the covenant.

D. In the absence of express annexation of the benefit of a covenant, s 78 of the Law of Property Act 1925 implies annexation of the benefit of a covenant to the land of the covenantee.

A

Statement B is wrong.

The annexation of the benefit of a covenant attaches the benefit of the covenant to the land of the covenantee at the TIME OF CREATION OF THE COVENANT.

Such annexation can be express in the deed in which the covenant is created or implied into the deed under s 78 of the Law of Property Act 1925.

Express assignment of the benefit of the covenant may take place at the time of the transfer of the land to a successor covenantee.

42
Q

One of the conditions for the benefit of a covenant to pass automatically at common law is that both the original covenantee and the person now seeking to enforce the covenant had/have a legal estate in the land.

Is the following statement True or False?

The person seeking to enforce the covenant must have the same legal estate in the land as the original covenantee.

A

the statement is False.

In Smith & Snipes Hall Farm Ltd v River Douglas Catchment Board [1948] 2 KB 500, the Court of Appeal considered that as s.78 of the Law of Property Act 1925 provided for a covenant relating to the land of the covenantee to be deemed to be made not only with the covenantee’s successors in title, but also with “persons deriving title under him or them”, so tenants holding a leasehold estate are included.

43
Q

X has a large estate of land and they sell a plot to A. A enters into two covenants with X: (a) not to use the land for any purpose other than a residence, and (b) to erect and maintain a fence on the boundary between the plot sold to A and X’s remaining land.

A starts to use the house for a bed and breakfast business.

Which ONE of the following remedies is not available?

A. Damages

B. Injunction

C. Specific Performance

A

statement C is correct.

Specific performance is not an available remedy because X does not want A to take a positive action to comply with the covenant; they want to prevent A from breaching a restriction, so an injunction to stop the breach of covenant and/or damages are the appropriate remedies.

44
Q

X has a large estate of land and they sell a plot to A. A enters into two covenants with X: (a) not to use the land for any purpose other than a residence, and (b) to erect and maintain a fence on the boundary between the plot sold to A and X’s remaining land.

Which ONE of the following statements most accurately describes the legal relationship between A and X?

A. Privity of estate exists between A and X.

B. Privity of contract exists between A and X.

C. Both privity of contract and privity of estate exist between A and X.

A

B is the correct answer

This question relates to freehold covenants, so privity of contract exists between A and X. Privity of estate only exists between landlord and tenant.

45
Q

X has a large estate of land and they sell a plot to A. A enters into two covenants with X: (a) not to use the land for any purpose other than a residence, and (b) to erect and maintain a fence on the boundary between the plot sold to A and X’s remaining land.

X now sells the remainder of their estate to Y and A sells their property to B.

Which ONE of the following statements most accurately describes the action Y can take if B breaches the covenants?

A. Y can sue B as privity of contract exists between them.

B. Y can sue B in respect of the covenant not to use the land for any purpose other than a residence, because the burden of the covenant passed to B.

C. Y can sue B in respect of both covenants (a) and (b) because the burden of both covenants passed to B.

A

B is correct.

Whilst at common law the burden of the covenants will not pass to B, in equity Y obtains the benefit of the covenants and B takes the burden of covenant (a) (the restrictive covenant) under Tulk v Moxhay, but not (b) which is a positive covenant.

46
Q

One of the conditions that has to be satisfied for the burden of a covenant to run in equity is that the person against whom it is sought to enforce the covenant must have notice of it.

Select the correct phrase from the list below to complete the following sentence:

Where the burdened land is registered, a restrictive covenant will be protected by the entry of a notice on the Charges Register of the title affected as it is

A. An interest affecting a registered estate.

B. A registrable disposition.

C. An overriding interest.

A

A is correct.

A restrictive covenant is classified in registered land as an interest affecting a registered estate.

47
Q

In which one or more of the following ways can a covenant over freehold property be discharged?

To obtain credit for this question you must identify all correct statements.
Hide answer choices

A. By implied consent.

B. By application to the High Court.

C. By application to the Lands Tribunal.

D. By application to the Local Authority.

E. By express agreement.

A

Statements C and E are correct

By application to lands tribunal and by express agreement

(Note: from 1 June 2009 the Lands Tribunal became the Lands Chamber of the Upper Tribunal, but is still referred to as the Lands Tribunal).

48
Q

A barrister is the freehold owner of two houses. He lives in one house (‘the Retained Land’). He sells the other house (‘the Property’) to an architect. The transfer contains the following clause:

“For the benefit and protection of the Retained Land the Buyer and his successors in title covenant with the Seller and his successors in title to only use the Property as a private dwelling house”

The architect sells the Property to a doctor. The barrister sells the Retained Land to a vet. The doctor sells the Property to a friend.

Which ONE of the following statements best describes the parties to the various transactions?

A. The barrister is the original covenantee and the vet owns the land burdened by the covenant.

B. The barrister is the original covenantor and the friend owns the land burdened by the covenant.

C. The architect is the original covenantee and the friend owns the land which benefits from the covenant.

D. The architect is the original covenantor and the vet owns the land which benefits from the covenant.

E. The architect is the original covenantor and the barrister owns the land which benefits from the covenant.

A

The correct option is D.

The architect is the original covenantor. Option C is, therefore, wrong.

The friend now owns the land burdened by the covenant – the Property.

The original covenantee was the barrister. Option B is, therefore, wrong.

The vet now owns the land that benefits from the covenant. Option A is, therefore, wrong.

Option E is wrong as, although the architect is the original covenantor, the barrister no longer

49
Q

A client owns a freehold property with a registered title. When the client bought the property, their solicitor advised them that the property had the benefit of two covenants over a piece of adjoining land. The covenants were entered into when the previous owner of the client’s property sold off the adjoining land to a developer.

The two covenants provided that the developer was required to build and maintain a fence on the adjoining land and that the developer would not be allowed to use the adjoining land for business purposes. The covenants were given by the developer on behalf of himself and his successors and with the intent that the burden of the covenants would run with the adjoining land. Both covenants were registered in the Charges Register of the title to the adjoining land immediately following the sale to the developer.

The developer sold the adjoining land six months ago to a dentist who has just started running her dental practice from the adjoining land. She is also refusing to maintain the fence. The title to the adjoining land reveals that the dentist agreed to observe the two covenants and indemnify the developer if the developer suffered any loss as a consequence of the dentist’s breach of covenant.

Which ONE of the following statements best describes the action that the client can take against the dentist?

A. The client can sue the dentist for breach of both covenants because the burden of both covenants has passed to the dentist.

B. The client can sue the dentist only for breach of the covenant not to use the land for business purposes.

C. The client cannot sue the dentist for breach of either covenant because the covenants were expressed to be personal to the developer.

D. The client cannot sue the dentist for breach of either covenant because the dentist was not a party to the original covenants.

E. The client can sue the dentist for breach of both covenants because the dentist gave an indemnity covenant to the developer.

A

Option B is correct.

The burden of a covenant can only pass (in equity) if it meets the requirements laid out in Tulk v Moxhay: it is restrictive, it was made to benefit land retained by the covenantee, it touches and concerns the covenantee’s land, there was an intention that the burden would run with the burdened land and the owner of the burdened land had notice of the covenant. The covenant not to use the land for business purposes satisfies all these requirements.

The covenant to build and maintain the fence is positive, however and so the burden of that covenant does not pass (despite its registration). It cannot therefore be enforced directly against the dentist.

Option A is wrong. As explained above, although the burden of the covenant not to use the land for business purposes passes to the dentist, the covenant to maintain the fence is positive and the burden of that covenant does not pass.

Option C is wrong. A covenant that is expressed as personal would not touch and concern the covenantee’s land but the facts indicate that this covenant was given by the developer on his own behalf and on behalf of his successors in title. It was not, therefore, expressed to be personal to the developer.

Option D is wrong because, as discussed at Option B above, the burden of a restrictive covenant does run with the land if the requirements laid out in Tulk v Moxhay are satisfied. The client can sue the dentist for breach of the restrictive covenant even though the dentist was not a party to the original covenant. Not being a party to the original covenant would, however, preclude the direct enforcement of the positive covenant against the dentist.

Option E is wrong. Although we are told that the dentist gave an indemnity covenant to the developer, this simply means that if the client sues the developer, then the developer can in turn sue the dentist under the indemnity covenant. An indemnity covenant given by a successor covenantor does not allow direct action to be taken against them. The client can still sue the developer directly because the client has the benefit of both of the covenants and the developer is still liable as the original covenantor.

50
Q

The freehold owner of a farm (the seller) sells a field containing a barn forming part of the farm (‘the Property’) to a buyer. The transfer to the buyer contains the following:

“The Buyer covenants with the Seller to repair and maintain the barn forming part of the Property”

Which ONE of the following statements best describes the right created by the seller and the buyer?

A. The positive covenant is a legal interest as it has been created by deed.

B. The restrictive covenant is an equitable interest although it has been created by deed.

C. The positive covenant is an equitable interest although it has been created by deed.

D. The restrictive covenant is a legal interest as it has been created by deed.

E. This creates an easement in favour of the buyer to access the barn forming part of the Property.

A

The correct option is C.

A covenant is not capable of being legal (s 1(3) LPA 1925). Creation by deed does not alter this. Options A and D are, therefore, wrong.

The covenant in question is positive as it will require effort or expenditure to perform it. Option B is, therefore, wrong.

Option E is wrong as a covenant has clearly been created, not an easement.

51
Q

A market research analyst is the freehold owner of two houses. The analyst sells one house (‘the Property’) to an occupational therapist. The transfer to the occupational therapist contained the following clause:

“The Buyer and her successors in title covenant with the Seller and his successors in title to use the Property only as a private dwelling house”

Elsewhere in the transfer, the other house is identified as the seller’s ‘Retained Land’. The occupational therapist has sold the Property to a recruitment consultant. The market research analyst has sold his house to a teacher. The recruitment consultant has started running a business from the Property in breach of the covenant.

Which ONE of the following statements best describes the basis upon which the teacher has the benefit of the covenant?

A. There has been an express assignment of the benefit of the covenant to the teacher.

B. There is statutory annexation as the land to be benefited is identified in the transfer.

C. The benefit passes to the teacher at common law as the criteria are met and so the teacher can pursue a claim against the recruitment consultant.

D. There is express annexation as words of annexation have been used.

E. There is a mutual benefit and burden enabling the teacher to enforce the covenant.

A

The correct option is B.

There is nothing in the facts to suggest that there has been express assignment. Option A is wrong.

There is no express annexation as words of annexation have not been used and the land to benefit from the covenant is not identified in the clause creating the covenant. Option D is wrong.

Option C is accurate as the benefit will have passed to the teacher at common law. However, at common law, the burden of covenants does not run and if (as here) the covenant is restrictive then the burden will only pass in equity. In order for the teacher to pursue a claim, they must show that benefit of the covenant has passed to them in equity . Option C is, therefore, not the best answer.

There is nothing on the facts to suggest a mutual benefit and burden. This only applies to positive covenants and here the covenant is restrictive. Option E is, therefore, wrong.

For these reasons, the only way in which the benefit of the covenant can pass to the teacher is by way of statutory annexation.

52
Q

A solicitor is acting on behalf of a buyer of a registered freehold property. The seller’s solicitor has informed the buyer’s solicitor that the land has the burden of an obligation not to use the property as anything other than a private dwelling house.

What will the buyer’s solicitor be looking for when examining the Land Registry official copy of the title to the property to confirm that the obligation is properly registered?

A. An entry referring to a restrictive covenant in the Property Register.

B. An entry referring to a positive covenant in the Property Register.

C. An entry referring to an easement in the Charges Register.

D. An entry referring to a restrictive covenant in the Charges Register.

E. An entry referring to a positive covenant in the Charges Register.

A

Option D is the correct answer.

The obligation in question is a restrictive covenant (not positive) and therefore B, C and E must be wrong. The burden of restrictive covenants appears as a notice in the charges register and not the property register and so A is wrong.

53
Q

An entry on the Charges Register of a registered freehold title reveals a notice of a covenant. The covenant states that the land can only be used for residential purposes and it is not expressed as being personal to the original parties. The covenant also states that it is intended to benefit the neighbouring freehold title, which is also a residential property. The wording of the covenant, however, makes no mention of the intention for the burden to run with the land.

Does the burden of this covenant run with the land?

A. No, because the covenant does not touch and concern the benefited land.

B. Yes, because the covenant is positive and all the requirements for the burden to run with the land have been satisfied.

C. No, because the intention for the burden of the covenant to run with the land was not expressly stated when the covenant was created

D. Yes, because all the requirements for the burden to run have been satisfied.

E. Yes, because registration of the covenant is confirmation from Land Registry that all the requirements for the burden to run have been satisfied.

A

Option D is the correct option.

The covenant is restrictive, there appears to have been identifiable land owned by the covenantee at the time the covenant was created, the covenant touches and concern the benefited land (see below), the intention for the burden to run will be implied by s79 LPA 1925 (as it has not been expressly shown) and the owner of the burdened land has notice of the covenant through its registration. Consequently, the burden of the covenant does run with the land.

Option A is wrong because the covenant does “touch and concern” the land. The covenant benefits only the owner of the benefited land for the time being (if separated from the land, it would cease to be advantageous to them), it affects the value of the benefited land (the value could drop if the burdened land was not used for residential purposes) and the covenant is not expressed to be personal.

Option B is wrong because this covenant would be interpreted as being restrictive in nature despite being worded in a positive fashion. It is the substance and not the wording which determines the nature of the covenant.

Option C is wrong because the intention for the burden of a covenant to run with the land can be implied by virtue of s79 LPA 1925, as indicated above in the explanation supporting Option D.

Option E is wrong because registration of the covenant does not of itself confirm that the burden of it would run with the land.

54
Q

The owner of a freehold farmhouse, who completed the purchase of the property six months ago, has started to use one of the outbuildings for her printing business. Her neighbour, the current freehold owner of the adjoining field, has now complained that the noise of the printing press disturbs the cattle grazing in the field and has produced a copy of the transfer deed entered into three years ago when the previous owner of the farmhouse sold off the field to its previous owner. The transfer deed included the following wording:

“The Seller with the intent and so as to bind the retained land [the farmhouse and outbuildings] and to benefit and protect the land hereby transferred to the Buyer [the field] hereby covenants with the Buyer that he and his successors in title will use the retained land for domestic and agricultural use only.”

The “Seller” and the “Buyer” were defined in the transfer deed as including successors in title. A notice recording the wording above was registered in the register of title of the farmhouse at the time. The owner of the farmhouse had no knowledge of this deed as she did her own conveyancing.

Can the neighbour take action to stop the owner of the farmhouse using the outbuilding for her printing business?

A. Yes, because the covenant has been registered on the register of title of the farmhouse.

B. Yes, because the benefit and burden of all freehold covenants automatically pass with the land.

C. No, because it is a positive covenant and the burden does not pass with the land.

D. No, because the owner of the farmhouse must have actual knowledge of the covenant to be bound by it.

E. No, because although the burden of the covenant has passed, only the previous owner of the field has the ability to take action to enforce it.

A

Option A is correct

This is a restrictive covenant and the benefit and burden can pass to successors of the original parties to the covenant. The requirements for the benefit and the burden of the covenant to pass automatically have been satisfied. The wording in the transfer deed means that the benefit of the covenant has been expressly annexed and shows an intention for the burden to run. The facts shows that the covenant was registered on the title to the farmhouse before the date of registration of the current owner and so the current owner had notice.

Option B is wrong - it is not true to say that the benefit and burden of all freehold covenants will pass. As explained above, certain requirements must be satisfied for the benefit and burden of freehold covenants to pass with the land (including registration of the covenant on the title register of the burdened land in registered land) and only the burden of restrictive covenants can directly pass to the successors of the burdened land.

Option C is wrong - this is not a positive covenant.

Option D is wrong - actual knowledge of the covenant is not required in order to bind the buyer of the burdened land – the registration of the notice on the register of title of the farmhouse amounts to notice of the covenant to the current owner.

Option E is wrong - the benefit of the covenant does not remain solely with the original covenantee and has passed to the current owner of the field.

55
Q

A solicitor is acting for a buyer of a house. The house has a registered freehold title and the seller is a sole registered proprietor. The seller’s solicitor tells the buyer’s solicitor that the house is subject to a covenant which states that it must only be used for residential purposes. The buyer also tells his solicitor that when the seller’s mother showed him around the house, she commented that the house had been a good investment for her and her son and she hoped it would be for the buyer too.

Which ONE of the following statements best explains whether the buyer may be bound by the issues affecting the title to the house?

A. The buyer will only be bound by the positive covenant if it is noted in the Charges Register of the title, however he may be bound by any trust interest which the seller’s mother has in the house even if it is not protected by an entry on the title register.

B. The buyer will not be bound by the positive covenant even if it is noted in the Charges Register of the title and he will only be bound by any trust interest which the seller’s mother has in the house if there is a restriction in the Proprietorship Register to protect it.

C. The buyer will be bound by the restrictive covenant if it is noted in the Charges Register of the title and he will also be bound by any trust interest which the seller’s mother has in the house if it is protected by an entry on the title register.

D. The buyer will be bound by the restrictive covenant if it is noted in the Charges Register of the title and he may also be bound by any trust interest which the seller’s mother has in the house even if there is no entry on the register in respect of it.

E. The buyer will be bound by the restrictive covenant if it is noted in the Charges Register of the title, but he will only be bound by any trust interest which the seller’s mother has in the house if there is a restriction in the Proprietorship Register to protect it.

A

D is correct

despite being worded in a positive way, the substance of the covenant is restrictive, so A and B are wrong.

In the registered system a restrictive covenant must be protected by a notice on the Charges Register in order to bind successors in title.

As regards the seller’s mother, she may have a trust interest if she invested money in the house. This trust interest may be overriding if she is also in actual occupation of the house and the conditions of paragraph 2 of Schedule 3 of the Land Registration Act 2002 are satisfied.

Overriding interests bind even though there is no mention of them on the registered title therefore, option C is wrong.

Finally, whilst a beneficiary can register a restriction on the Proprietorship Register, this does not protect the trust interest, instead, it alerts a potential buyer of the need to overreach the trust interest. So option E is wrong.

56
Q

Which one or more of the following must be satisfied for there to be a valid legal express easement over registered land?
Hide answer choices

A. Must be created by deed under s52 LPA 1925

B. Must be created by signed writing under s53(1)(a) LPA 1925

C. Must satisfy s1(2) LPA 1925 ie be for a period equivalent to a freehold or a leasehold estate

D. Must fall within one of the conditions in Sch 3 para 3 LRA 2002

E. Must be registered under s27 LRA 2002, as it is a registrable disposition

A

Options A, C and E are correct.

To be legal, an expressly created easement must satisfy s1(2) LPA 1925 as to duration, be created by deed (s52 LPA 1925) and be completed by registration (s27 LRA 2002).

Registration also makes the easement enforceable against a buyer of the land.

Sch 3 para 3 LRA 2002 only applies to easements created impliedly on a sale of part or by prescription and deals with enforceability against a buyer, not whether the easement is legal in the first place.

57
Q

Legal easements can be acquired in several ways.

Which one of the following statements is WRONG?

A. A legal easement can be acquired by statute.

B. A legal easement can be acquired by express or implied grant.

C. A legal easement can be acquired by oral agreement.

D. A legal easement can be acquired by presumed grant or prescription.

E. A legal easement can be acquired by express or implied reservation.

A

Option C is wrong. To create an express legal easement it must be granted by deed.

58
Q

For the last 45 years Megarry, freehold owner of Serjeants’ Close, has maintained a greenhouse on the boundary with Tree Court, and has used a convenient short cut over Tree Court.

The freehold of Tree Court has just been bought by Wade who proposes to fill the site with a multi-storey car park.

In which one of the following ways can Megarry claim a right of way over Wade’s land?

A. By implied grant.

B. By prescription.

C. By deed.

A

Option B is correct.

Megarry has used the right of way for 45 years and may successfully claim that he has the benefit of an easement by prescription.

Megarry may have acquired an easement by lost modern grant already or he can claim under both periods in s.2 Prescription Act 1832 (20 and 40 years) if he brings a court action.

The right is capable of being an easement and it has been enjoyed as of right, that is, without force, secrecy or permission. There seems to have been continuous use and the right is exercised by one fee simple owner against another.

There is no evidence that there has been a sale of part and so the easement has not be created by implication nor is there evidence of an express agreement between the parties by deed.

59
Q

For the last 45 years Megarry, freehold owner of Serjeants’ Close, has maintained a greenhouse on the boundary with Tree Court, and has used a convenient short cut over Tree Court.

The freehold of Tree Court has just been bought by Wade who proposes to fill the site with a multi-storey car park.

Megarry may also be entitled to a right to light.

Which ONE of the following statements most accurately describes the legal position?

A. Megarry has a right to light by implied consent.

B. Megarry can claim a right to light through prescription.

C. Megarry can claim a right to light through strict necessity.

D. Megarry cannot claim a right to light as it is too vague a benefit to be classified as an easement.

A

Option B is correct.

To “lie in grant” Megarry must claim the right to receive light through a specific opening or window. The amount of light that can be claimed as an easement is limited to that required for the use of the building for the ordinary purpose for which it was constructed.

A greenhouse is a building (see Allen v Greenwood) and is entitled to the amount of light needed to cultivate plants. Megarry will be able to claim under lost modern grant if he fulfils all the common law conditions.

If not, as an alternative, he could bring a court action under s3 Prescription Act 1832 if the right has been used for 20 years without written consent.

The claim cannot be defeated because of force, secrecy or by verbal consent and neither does the user need to be by one fee simple owner against another. This relaxation of the common law conditions only applies to easements of light.

There is no evidence that there has been a sale of part and so the easement has not be created by implication nor is there evidence of an express agreement between the parties by deed.

60
Q

Andrew owned a large freehold house which he converted into two semi-detached houses. He sold the freehold of one of the houses (House A) and retained the other (House B) as his dwelling-house. All the drains and sewers serving House A pass under the garden of House B. These were not mentioned in the transfer of House A.

Which ONE of the following statements most accurately describes the legal basis upon which the new owner of House A can claim to have an easement of drainage under House B?

A. Prescription or presumed grant.

B. Express grant.

C. Express reservation.

D. Implied Grant

E. Implied reservation.

A

Option D is correct.

The new owner of House A has acquired the rights as a result of implied grant.

You have been told that the transfer did not mention these rights so they have not passed to the owner of House A by virtue of an express grant and, as all the land was owned and occupied by Andrew prior to the sale, prescription cannot apply in this case as Andrew cannot acquire easements over his own land.

61
Q

Andrew owned a large freehold house which he converted into two semi-detached houses. He sold the freehold of one of the houses (House A) and retained the other (House B) as his dwelling-house. All the drains and sewers serving House A pass under the garden of House B. These were not mentioned in the transfer of House A.

By which one or more of the following implied grant methods was the buyer most likely to have acquired an easement of drainage?

A. Strict necessity.

B. Common intention.

C. Wheeldon v Burrows.

D. Section 62 Law of Property Act 1925

A

Options B, C and D are correct. Strict necessity is unlikely to apply as there would be alternative methods of drainage and sewerage (such as cess pits/soak-aways) and so the land could be used without the easement.

The use of the drains and the sewers may have been in the intention of the parties and so common intention could be available (Wong v Beaumont Property Trust Ltd) - if a common purpose if known to the parties, the right claimed is needed in order for the common purpose to be fulfilled.

Also, as Andrew used the drains before the sale of part, the right was a ‘quasi easement’ which is then converted into an implied easement in the sale of part transfer in favour of the buyer, provided that the use was continuous, apparent (by means of manhole covers, for example) and necessary for the reasonable enjoyment of the land.

S62 may also apply as, although there has not been diversity of occupation immediately before the sale of part, if the use is continuous and apparent (as it appears to be on the facts), s62 will have the effect of converting the right or privilege into an easement.

62
Q

Andrew owned a large freehold house which he converted into two semi-detached houses. He sold the freehold of one of the houses (House A) and retained the other (House B) as his dwelling-house. All the drains and sewers serving House A pass under the garden of House B. These were not mentioned in the transfer of House A.

Is the following statement is True or False:

If Andrew had sold House B instead of House A, he would have been entitled to an easement of drainage by implied reservation.

A

the statement is True.

As a seller on a sale of part Andrew can only rely on the implied reservation methods of strict necessity or common intention. Although, it is unlikely strict necessity would apply, it seems more likely common intention might work here (for the reasons set out in Question 7).

A seller cannot rely on Wheeldon v Burrows because the court will not allow the seller to ‘derogate from his grant’.

63
Q

Which one or more of the following statements are correct?
Hide answer choices

A. A legal easement, however created, must be registered as a land charge to bind a purchaser of land with unregistered title.

B. An expressly created legal easement may be protected by registration under section 27 LRA 2002 or as an overriding interest under Schedule 3 paragraph 3 LRA 2002 against a purchaser of a registered title.

C. Schedule 3 paragraph 3 LRA 2002 only protects easements created over land with registered title by implication on a sale of part or by prescription.

D. Expressly created equitable easements must be protected by a notice on the register in registered title, or registered as a D(iii) land charge in unregistered title, in order to bind a purchaser.

E. Until registration, an expressly created easement by deed over registered land remains equitable only.

A

Options C, D and E are correct.

All legal easements, however created, bind the world in the unregistered system. As we saw in question 1, in the registered system an expressly created legal easement must be protected by registration and can never be overriding.

64
Q

The freehold owner of a detached house (‘the Property’) is concerned about the extension proposed by her new neighbour. The proposed extension will make parts of the Property’s garden very gloomy.

Does the Property have an easement?

A. Yes, the Property has an easement of light for the garden but must show an infringement.

B. Yes, there is a clear easement of light which would prevent the new neighbour’s development.

C. No, the Property does not have an easement as no new negative easements can be created.

D. No, the Property does not have an easement of light as there is no defined aperture in this garden.

E. Yes, the Property has an easement to have an uninterrupted view from her garden and can stop the new neighbour’s extension.

A

The correct Option is D.

The right to light is a recognised negative easement (therefore, Option C is wrong) but there is no general right to light (therefore, Options A and B are wrong).

A right to light must be via a defined aperture and it is hard to see how this would be a garden generally without a specific aperture benefiting from the right.

Option E is wrong as there can be no easement for an uninterrupted view.

64
Q

Alice owned the freehold of a factory and in January 2010 granted a ten year lease by deed of part of the building to Sarah. During the lease, Alice allowed Sarah to park her car in the courtyard outside the factory. The courtyard was owned by Alice and was not included in the lease. In February 2020 Alice granted Sarah a further ten year lease by deed on exactly the same terms as the original lease, without mentioning the agreement as to parking.

Which ONE of the following statements is correct?
Hide answer choices

A. Sarah has an oral easement to use the courtyard to park.

B. Sarah has an implied easement to use the courtyard under section 62 LPA 1925.

C. Sarah has no right to park as it was not expressly included in her renewed lease.

D. Sarah has an easement by prescription as she has used the courtyard since 2010.

A

Option B is correct.

The second lease in February 2020 counts as a sale of part and Sarah’s pre-existing licence to park can be elevated to the status of an easement (see Wright v Macadam) as it had all the necessary characteristics of a valid easement, by being implied into the lease deed.

Easements cannot be created orally and prescription will not work, as she has not used the right for 20 years and has been using it with permission, hence not satisfying the common law conditions.

65
Q

Last year, a client bought a registered freehold cottage, without consulting a solicitor. The seller had built the cottage 20 years ago and occupied it from then until completion of the sale. The seller also retained some land between the cottage and the main road. The transfer deed to the client made no reference to a right to use the drains serving the cottage which run under the seller’s retained land and which connect to the sewer under the main road. Manhole covers on the client’s land indicate the presence of the drains. The seller has now sold the retained land and the new owner is disputing the client’s right to use the drains.

Which of the following statements best explains how the client may claim an easement of drainage in these circumstances?

A. As there was diversity of occupation at the time of the transfer to the client, the client may claim an implied easement under section 62 of the Law of Property Act 1925.

B. The client may claim an implied easement of necessity, as the cottage cannot be used without a right of drainage.

C. The client may claim an implied easement under the rule in Wheeldon v Burrows.

D. The client may claim a prescriptive easement at common law.

E. The client may claim a prescriptive easement under the doctrine of lost modern grant.

A

Option C is correct because the rule in Wheeldon v Burrows will imply an easement on a sale of part where there is a continuous and apparent quasi-easement which is necessary for the reasonable enjoyment of the dominant tenement and which is in use at the time of the sale of part of the dominant tenement. All of these elements are present on the sale to the client.

Option A is wrong as there was no diversity of occupation at the time of the transfer to the client. The cottage was owned and occupied by the seller. In any event, diversity of occupation is only needed to create an easement under s62 LPA where there is a licence or privilege in favour of the dominant land, which then becomes an easement on completion of the sale of that dominant land. (Note that an easement may have been created under s62 LPA 1925 in the circumstances in this question as there was a continuous and apparent quasi easement in use at the time of the sale for the benefit of the land being sold – see P&S Platt Ltd v Crouch and Wood v Waddington). It is, however, the reference to there being diversity of occupation at the time of the transfer that makes this option wrong).

Option B is wrong because an easement of drainage is not a strict necessity here. It is possible to use the cottage without the easement of drainage by installing a septic tank or cesspit.

Option D is wrong because a common law prescriptive easement is based on a presumption of continuous use since 1189 where the easement has been used continuously for 20 years. However the presumption is rebuttable here because the cottage was only built 20 years ago. It can therefore be confirmed that use of the easement did not run all the way back to 1189.

Option E is wrong as the period of the seller’s use is not relevant here when deciding if there is a prescriptive easement. The client would need to use the drains for 20 years before such an easement could be considered.

65
Q

A landowner owns a field (which is registered at Land Registry) abutting a main road. A buyer buys part of the field which has no direct access to the main road. The landowner agrees to grant the buyer a right of way over the part of the field the landowner still owns.

Which ONE of the following statements represents the best advice to ensure that the buyer will have an indefeasible right of way over the landowner’s part of the field to gain access to the main road?

A. It will be necessary to document this arrangement which may be done by a deed from the landowner to the buyer. Only the landowner’s signature is needed.

B. It will be necessary to document this arrangement which may be done by a deed from the landowner to the buyer. Only the landowner’s signature is needed but his signature must be witnessed by a third party and the buyer must register the deed at the Land Registry.

C. It will not be necessary to document this arrangement. The landowner’s word is sufficient.

D. It will be necessary to document this arrangement which may be done by a deed from the landowner to the buyer. Only the landowner’s signature is needed but his signature must be witnessed by the buyer.

D. It will be necessary to document this arrangement which may be done by a deed from the landowner to the buyer. Only the landowner’s signature is needed but his signature must be witnessed by a third party.

A

Option B is the best answer. For the buyer to have an indefeasible easement the easement must be expressly created by deed and registered at the Land Registry.

It is not possible to grant a legal easement orally so Option C is wrong.

Option A is wrong because for a deed to be valid an individual’s signature must be witnessed.

Option D is wrong because the witness must be a third party.

Option E is correct but for the easement to retain its legal status it must be registered as a registrable disposition at the Land Registry.

66
Q

In 1999, a man purchased a freehold property consisting of a farm house and outbuildings including a large separate barn (‘the Barn’). The Barn has its own access to the main road, but the man has always used a track (‘the Track’) running past the main house to access the Barn. He prefers the Track as it avoids a dangerous right-hand turn. Last year, the man sold the Barn to a woman. The transfer included an express right over the main point of access but made no mention of the Track.

Which of the following answers best describes how an easement over the Track has been created?

A. This is an easement of necessity which allows the woman to access the Barn.

B. This is a common intention easement and the woman can only fulfil the purpose with the easement.

C. The easement is created by Wheeldon v Burrows as a quasi-easement in use at the time of the sale.

D. The easement was expressly created in the transfer to the woman.

E. The easement was created by prescription as it has been used for over 20 years.

A

The correct Option is C.

This cannot be an easement of necessity as there is an alternative means of access and therefore, the Barn is not landlocked. Option A is, therefore, wrong.

There is no indication of a common purpose or intention in the facts, so Option B is wrong.

The transfer to the woman contained an express grant only in relation to the main point of access, not the Track. Option D is, therefore, wrong.

This cannot be an easement by prescription as, although the use started in 1999, it could not become an easement until there was a separate dominant and servient owner. This only happened when the Barn was sold last year. Option E is, therefore, wrong.

The man exercised a quasi-easement over his own land. On the transfer to the woman, this became a legal easement on the basis that the easement was clearly continuous and apparent, necessary for the reasonable enjoyment of the land (to avoid the busy junction) and in use at the time of the sale. Option C is, therefore, correct.

67
Q

A freehold owner granted a 25-year lease of their property (‘the Property’). Five years after the lease was created, the freehold owner gave informal permission for the tenant to erect a sign upon the freehold owner’s adjoining land advertising the tenant’s business. Last month, when the lease came to an end, the tenant purchased the freehold to the Property. The transfer made no mention of the sign.

Which of ONE the following statements best describes how an easement in relation to the sign has been created?

A. The easement was created by s 62 LPA 1925 from an existing privilege upon the transfer of the freehold to the tenant.

B. The easement was expressly created on the transfer of the freehold to the tenant.

C. This is an easement of necessity which allows the tenant to erect a sign.

D. The easement was created by prescription as it has been used for over 20 years.

E. The right is not capable of existing as an easement as it does not accommodate the dominant tenement.

A

The correct Option is A.

The easement in relation to the sign was created by implication by s 62 LPA 1925. There was an existing privilege (the freehold owner’s informal permission to erect the sign), there was diversity of occupation (the freehold owner owning the servient land and the tenant occupying the Property) and there was a conveyance (the transfer of the freehold to the tenant). The effect of the conveyance is to create a legal easement in relation to the sign.

There is no mention of the easement in the transfer of the Property. Option B is, therefore, wrong.

Although the easement has been exercised for over 20 years, most of this period was whilst the Property was let. A prescriptive right can only be created as between two freehold owners. Option D is, therefore, wrong.

It is hard to consider any circumstances in which an easement relating to a sign could be an easement of necessity. Option C is, therefore, wrong.

Option E is wrong as the right is capable of being an easement as it does accommodate the dominant tenement.

68
Q

A client has purchased a plot of land from a seller. The plot forms part of the garden to the seller’s house. The seller told the client that they built their house ten years ago and that they intended to continue living in the house (the “retained land”) following the sale of the plot to the client.

The client has now decided that she wants to construct a dwelling house on her plot but she has discovered that drains serving the seller’s retained land run across the plot. The seller has refused to divert the drains. There was nothing about the drains in the transfer to the client.

Can the client force the seller to divert the drains so that they no longer cross the client’s plot?

A. Yes, because the seller is unlikely to have an easement of drainage implied by necessity.

B. No, because the seller has an easement of drainage implied by common intention.

C. Yes, because the seller has only used the drains for 10 years therefore he cannot claim an easement by prescription.

D. No, because it is likely the seller has an easement of drainage implied under the rule in Wheeldon v Burrows.

E. Yes, because the seller does not appear to have expressly reserved the right to use the drains in the transfer.

A

Option B is correct because the seller is likely to have impliedly reserved an easement by common intention. The client knew that the seller intended to continue to live in the house and therefore the seller’s easement of drainage would be necessary to give effect to the intended use of the seller’s retained land. The seller does not have to divert the drains.

Option A is wrong. A right of drainage is unlikely to be considered an easement of necessity but as Option B indicates, an easement can be implied by common intention and the seller does not have to divert the drains.

Option C is wrong. Whilst the seller will not have an easement by prescription (because she has not used the drains for at least 20 years). As Option B indicates, the seller will be likely to have an easement by common intention so the seller does not have to divert the drains.

Option D is wrong because it is not possible to have an implied reservation of an easement under the rule in Wheeldon v Burrows. It only works to grant easements to buyers (and their successors) on a sale of part.

Option E is wrong because whilst there appears to be no express reservation, as Option B indicates, an easement of drainage is implied by common intention and the seller will not have to move the drains.

69
Q

A man has a pedestrian and vehicular right of way over a privately owned farm track leading from his house to the public highway. Several other local residents also have permission to use the track to access their properties from the public highway. The deeds creating the rights of way are silent as to obligations to repair the track on the part of either the man, the farmer who is the freehold owner of the track, the other local residents or anyone. The track does not form part of the public highway.

The man has noticed that, through normal use by those entitled to use it, the track has developed a number of potholes and has generally fallen into disrepair such that it is now dangerous in parts to walk on and is damaging to vehicles using it. The man seeks advice as to who is obliged to maintain or repair the track.

Which ONE of the following statements best describes who must maintain and repair the track?

A. The freehold owner.

B. Nobody.

C. The man.

D. The man and the other local residents who use the track.

E. The man, the other local residents who use the track and the freehold owner.

A

Option B is correct. Once a right of way exists, the servient owner (the owner of the land over which the easement exists) is under no obligation to maintain or repair it. Similarly, the dominant owner (the owner of the land who can exercise the rights attaching to the easement) has no obligation to maintain or repair the right of way.

The liability to maintain or to repair the right of way once constructed primarily depends upon the terms of the grant or reservation. In the absence of express stipulation or special circumstances, neither the servient owner nor the dominant owner is liable to maintain or repair it.

In this situation, there is no express stipulation nor any special circumstances and so neither the freehold owner as servient owner, nor the man/the other local residents as dominant owners are liable or obliged to repair the track.

Accordingly, Option B is correct and the other options are necessarily wrong.

70
Q

Which statutory provision(s) govern the creation of legal mortgages?

A. Schedule 3 paragraph 2 Land Registration Act 2002.

B. Section 2 Law of Property Act 1925.

C. Sections 85 and 86 Law of Property Act 1925.

D. Section 2 Law of Property (Miscellaneous Provisions) Act 1989.

A

Option C is the correct answer. Section 85 provides that a mortgage may be created by way of a charge by legal mortgage over freehold land (the main method of creation nowadays) and section 86 over leasehold land.

71
Q

Which ONE of the following statements accurately describes the extra requirement that exists in the case of legal mortgages of registered title?

A. The mortgage must be registered in the Charges Register of the mortgagor’s title.

B. The mortgage must be registered at the Central Land Charges Department as a C(iv) Land Charge.

C. The mortgagee should register the mortgage, so that it becomes an overriding interest.

D. The mortgagor should notify the mortgagee that the mortgage has become an overriding interest.

A

Option A is correct. A legal mortgage over a registered title is a registrable disposition, so for the mortgage to be a legal mortgage it must be completed by registration - s27 LRA 2002.

72
Q

Hector buys a house with the aid of a mortgage by deed from the Dalton Finance Company. Later he takes out a further mortgage by deed with the Stretton Building Society.

Is the following statement True or False?

As the title to the property is registered, the second mortgage should be registered at Land Registry.

A

The statement is True.

The second mortgage should be protected in exactly the same way as the first mortgage i.e. protected as a registrable disposition in the Charges Register of Hector’s title. Failure to take this step will result in the mortgage being treated as an interest affecting a registered estate (only recognised by equity) which would still require protection by registration of a notice (s32 LRA 2002).

73
Q

In 2012 David purchased a registered freehold house at 8 Orchard Road, Greenwich. Shortly after becoming the owner David created a mortgage by deed over the property in favour of the Best Building Society to secure a loan of £100,000.

In 2016 David created a second mortgage by deed over number 8 in favour of the Midshire Bank (“the Bank”) to secure a debt of £70,000. The Bank registered its mortgage against the title to the property. Later that year, David created a further mortgage by deed in favour of his friend Paul, who also registered his charge against the title.

David has not made any payments to the Bank for the last six months.

If the Bank wants to exercise its power of sale, which one or more of the following statements reflects its position?

A. A sale by a legal mortgagee takes effect subject to incumbrances which have priority over its mortgage, but free from incumbrances that it has priority over.

B. The Bank will be bound by Paul’s mortgage and will therefore be unable to sell.

C. The Bank will sell free of Paul’s mortgage as it was registered after the Bank’s was registered.

D. The Bank must obtain Paul’s permission in order for the sale to go ahead.

A

Options A and C are correct.

On a sale by a mortgagee the buyer takes the property subject to incumbrances that bind the mortgagee who is selling the property (s 104(1) LPA 1925). In this case the Bank’s mortgage is entered on the register before Paul’s mortgage, so it ranks before Paul’s mortgage (s 48 LRA 2002).

74
Q

In 2012 David purchased a registered freehold house at 8 Orchard Road, Greenwich. Shortly after becoming the owner David created a mortgage by deed over the property in favour of the Best Building Society to secure a loan of £100,000.

In 2016 David created a second mortgage by deed over number 8 in favour of the Midshire Bank (“the Bank”) to secure a debt of £70,000. The Bank registered its mortgage against the title to the property. Later that year, David created a further mortgage by deed in favour of his friend Paul, who also registered his charge against the title.

David has not made any payments to the Bank for the last six months.

Which ONE of the following statements correctly describes the position with regard to the Building Society’s mortgage?

A. As the Building Society’s mortgage is made by deed it will be binding on the Bank.

B. The Building Society’s mortgage will be binding on the Bank if it was entered on the register of the title to 8 Orchard Road before the date the Bank’s mortgage was entered on the register.

C. The Building Society’s mortgage will be binding on the Bank if it was registered as a C(i) Land Charge at the Central Land Charges Department before the date the Bank’s mortgage was registered.

D. The Building Society’s mortgage will not be binding on the Bank as the Bank had no notice of the Building Society’s prior mortgage.

A

Option B is correct.

A mortgage by deed must be registered against the title to become legal and to take priority over later mortgages (s 48 LRA 2002). In this case we need to know whether or not the Building Society protected its mortgage before the Bank’s mortgage was entered on the register.

75
Q

In 2012 David purchased a registered freehold house at 8 Orchard Road, Greenwich. Shortly after becoming the owner David created a mortgage by deed over the property in favour of the Best Building Society to secure a loan of £100,000.

In 2016 David created a second mortgage by deed over number 8 in favour of the Midshire Bank (“the Bank”) to secure a debt of £70,000. The Bank registered its mortgage against the title to the property. Later that year, David created a further mortgage by deed in favour of his friend Paul, who also registered his charge against the title.

David has not made any payments to the Bank for the last six months.

Which one or more of the following statements is/are correct?

A. The Bank cannot sell the property free of David’s interest as David is in occupation.

B. The effect of a sale by a mortgagee is to transfer the title of the mortgagor to the buyer, so David’s title will go to whoever the Bank sells the property to.

C. The Bank could apply to the court for a possession order to remove David from the property.

D. The Bank cannot obtain possession without the consent of the Best Building Society.

A

The correct answers are options B and C.

In order to transfer title to a new owner, the Bank does not technically need to take possession of the property.

If the Bank wants to sell the property with vacant possession, it will have to take possession of the property from David, or he will have to give up possession. If the Bank applies to the court for a possession order the court may postpone the order to give David time to repay the arrears. It is not necessary to obtain a court order, but it is safer to do so.

The benefit of obtaining a court order first is that the Bank will avoid any risk of committing an offence under the Criminal Law Act 1977.

76
Q

In 2012 David purchased a registered freehold house at 8 Orchard Road, Greenwich. Shortly after becoming the owner David created a mortgage by deed over the property in favour of the Best Building Society to secure a loan of £100,000.

In 2016 David created a second mortgage by deed over number 8 in favour of the Midshire Bank (“the Bank”) to secure a debt of £70,000. The Bank registered its mortgage against the title to the property. Later that year, David created a further mortgage by deed in favour of his friend Paul, who also registered his charge against the title.

Now consider Paul’s position. David created a further mortgage in favour of him in 2016. If possible, the Bank would like to sell the property free of Paul’s mortgage.

Which ONE of the following statements correctly reflects the position?

A. The Bank will be unable to sell free of Paul’s mortgage as it is a lower priority incumbrance than the Bank’s mortgage.

B. The sale will be subject to Paul’s interest as it is registered on the Charges Register of the title.

C. The sale is free of incumbrances with lower priority than the Bank’s mortgage, so Paul’s mortgage will not be binding on the purchaser.

A

Option C is correct - S104(1) LPA 1925. However the debt will still be owed by David to Paul

77
Q

In 2012 David purchased a registered freehold house at 8 Orchard Road, Greenwich. Shortly after becoming the owner David created a mortgage by deed over the property in favour of the Best Building Society to secure a loan of £100,000.

In 2016 David created a second mortgage by deed over number 8 in favour of the Midshire Bank (“the Bank”) to secure a debt of £70,000. The Bank registered its mortgage against the title to the property. Later that year, David created a further mortgage by deed in favour of his friend Paul, who also registered his charge against the title.

Assume that the property is sold.

On the basis that the Building Society has never registered its mortgage against the title, which one of the following statements most accurately describes the order of priority between the three mortgages?

A. The Bank first, followed by the Building Society and then Paul.

B. The Bank first, followed by Paul and then the Building Society.

C. The Building Society first, followed by the Bank and then Paul.

A

Option B is correct.

The Bank will rank first as its mortgage is entered on the register first. The Building Society’s mortgage is unenforceable against the Bank because the Bank is a purchaser for value (s 29 LRA 2002). As the Building Society never registered its mortgage, it will also rank behind Paul’s mortgage, as again Paul has registered (s48 LRA 2002).

78
Q

Is the following statement True or False?

An overriding interest must exist at the date a mortgage is registered at the Land Registry to be binding on the mortgagee under s.29 land Registration Act 2002.

A

The statement is False.

An overriding interest must exist at the date the mortgage is completed, not the date the mortgage is registered. s.29 LRA applies to Registrable Dispositions and Interests Affecting a Registered Estate. These must be registered before the date the mortgage is registered in order to bind a mortgagee.

79
Q

A solicitor acts for the owner of a freehold factory block (‘the Property’) subject to a mortgage (by deed) in favour of a bank (‘the Bank’). The Property is currently vacant since the owner’s business closed. The owner has not paid the mortgage payments for six months. The Bank believes the Property is worth £350,000 and the owner owes £370,000. The Bank would like to end the mortgage.

Which ONE of the following statements is the best approach for the Bank to pursue?

A. Take possession immediately with a view to redirecting income from the Property to the Bank.

B. Exercise the power of sale and pursue a debt action against the owner for any shortfall.

C. Pursue a debt action against the owner to recover the money owed to the Bank.

D. Seek an order for foreclosure to bring the mortgage to an end and vest title in the Property in the Bank.

E. Appoint a receiver to demand and receive income from the Property.

A

The correct Option is B.

Exercise of the power of sale will end the mortgage. As the value of the Property is lower than the debt, the Bank can pursue a debt action against the owner for the difference.

Taking possession, pursuing only a debt action and appointing a receiver do not end the mortgage. Therefore, Options A, C and E are wrong.

Foreclosure would end the mortgage. However, it is not in the Bank’s interest to foreclose as this would extinguish the mortgage and leave the Bank with no remedy to recover any shortfall from the owner. Option D is not therefore the best answer.

80
Q

Five years ago, the owner of registered freehold land created a first mortgage by deed over the land. Three years later, the owner granted a lease of the land for a term of six years, by deed. Last year, the owner created a second mortgage by deed over the land, which was then registered at the Land Registry. Two months later, the first mortgage was registered at the Land Registry. The legal date for redemption has passed in respect of both mortgages. The owner is four months in arrears of interest on the second mortgage.

Which ONE of the following statements best describes the exercise of the power of sale and the priority between the three interests?

A. The second mortgagee can exercise the power of sale and will be able to sell the land free from the lease, but subject to the first mortgage.

B. The second mortgagee can exercise the power of sale and will be able to sell the land free from the first mortgage, but subject to the lease.

C. The second mortgagee can exercise the power of sale and will be able to sell the land free from both the first mortgage and the lease.

D. The second mortgagee can exercise the power of sale and will have to sell the land subject to both the first mortgage and the lease.

E. As the second mortgage was created second in time, the second mortgagee will not be able to exercise the power of sale, but when the first mortgagee sells the land, the monies received will be applied first in discharge of the second mortgagee.

A

Option B is the best answer.

The power of sale in respect of the second mortgage has arisen, as the legal date of redemption has passed, and is exercisable, as there are arrears of interest. Accordingly, the second mortgagee can exercise the power of sale. The fact that the second mortgage was created second in time does not prevent the second mortgagee from exercising the power of sale. If the first mortgagee were to exercise the power of sale, it is correct that the proceeds of sale would be applied first in the discharge of the second mortgage under s105 LPA 1925. However, the power of sale has not arisen on the first mortgage.

Priority between mortgages is governed by the LRA 2002 and the first to appear on the register takes priority. The date of creation is irrelevant. The second mortgage was registered first and therefore has priority over the first mortgage and can sell free from it. Therefore, Option B is the best answer and Options A, D and E are wrong.

The lease is an overriding interest under Schedule 3 paragraph 1 LRA 2002, a legal lease for a term not exceeding 7 years. This will have priority over the second mortgage as it was in existence at the date of creation of the second mortgage. The second mortgagee will have to sell subject to the remaining term of the lease. Therefore, Option C is wrong.

81
Q

A borrower purchased a house (‘the Property’) four years ago with the assistance of a mortgage (by deed) with a bank (‘the Bank’). The borrower has been made redundant and has not paid the last mortgage payment. The Bank want to sell the Property immediately.

Which of ONE the following statements most accurately describes the position for the Bank?

A. The buyer need only check that the power of sale exists and has arisen.

B. The Bank can exercise the power of sale as it exists, has arisen and is exercisable.

C. The Bank cannot exercise the power of sale as the power of sale does not exist.

D. The Bank cannot exercise the power of sale as the power is not yet exercisable.

E. The buyer will take the Property subject to the mortgage in favour of the Bank.

A

The correct Option is D.

The power of sale exists (the mortgage was made by deed) and has arisen (the mortgage was made four years ago so the legal date for redemption will have passed). Therefore, Option C is wrong.

The power of sale is not exercisable as the borrower has only failed to pay one monthly payment, not two. Option B is, therefore, wrong.

Option A correctly states the legal position but does not answer the question and is, therefore, wrong.

Option E is wrong. A buyer would take free of the Bank’s mortgage upon the Bank exercising the power of sale.
Although the power of sale exists and has arisen, the power is not yet exercisable as there is not yet two months’ arrears. Option D is, therefore, correct.

82
Q

A solicitor acts for the freehold owner of a vacant office block (‘the Property’) subject to a mortgage (by deed) in favour of a bank (‘the Bank’). The owner purchased the Property five years ago on a 25-year mortgage. The owner has not paid the mortgage payments for the last two months after their tenant’s lease came to an end and vacated the Property. The Bank have told the owner that they plan to exercise their power of sale in relation to the Property. The owner wishes to oppose this as there is a downturn in the market and it is a dreadful time to sell.

Can the owner oppose the Bank in exercising its power of sale?

A. Yes, because the Bank must obtain a court order for possession prior to selling the Property.

B. No, because the power of sale is exercisable and the Property is not a dwelling house.

C. Yes, because the owner can apply to the court to adjourn the possession proceedings.

D. No, because the Bank can choose the timing of the sale but must obtain a possession order for possession.

E. Yes, because the Bank’s power of sale is not exercisable until the owner is in arrears for three months.

A

The correct Option is B.

The power of sale exists (the mortgage was created by deed), has arisen (the mortgage was created five years ago and the date for redemption should have passed) and is exercisable as the owner is in two months arrears of interest payments. Option E is, therefore, wrong as only two months of arrears are needed.

The Bank do not need to obtain a possession order as this is not a dwelling house and the Property is vacant. Therefore, Option A is wrong.

The owner cannot apply to the court to adjourn the proceedings as (i) there are no such proceedings and (ii) the Property is not a dwelling house. Option C is, therefore, wrong.

The Bank can choose the timing of the sale but do not need to obtain a court order for possession. Option D is, therefore, wrong.

83
Q

A lender has taken a mortgage over a property to secure a loan that it has made. The mortgage is by deed and has been duly registered at Land Registry. The borrower is several months late with its interest payments and the lender is considering possible remedies. The lender wishes to keep the mortgage in existence for the time being and to allow the borrower to remain in occupation and for the borrower to receive any income the property might produce.

Which of the following remedies is the most appropriate for recovering the money due whilst meeting the lender’s wishes?

A. Foreclosure

B. Taking possession

C. Appointing a receiver

D. Debt action

E. Sale

A

Option D is the best answer as a debt action would mean that the mortgage continued in force and the borrower remained in possession.

Options B and C are not the best answers. Taking possession would mean that the borrower no longer remained in occupation and appointing a receiver would mean that the borrower was no longer entitled to any income from the property.

Sale or foreclosure would bring the mortgage to an end and so, Options A and E are wrong.

84
Q

A client owns a registered freehold property. Two years ago, the client granted a lease, by deed, of an outbuilding on the property to a tenant for a period of five years. Last year, the client took out a loan from a building society. The loan was secured by a mortgage over the property. The mortgage was created by deed and was registered at Land Registry promptly following its completion. The client has not paid the instalments due on the mortgage for the last six months. The building society is planning to exercise its power of sale and has found a prospective buyer for the property. There is no mention of the lease on the register of title to the property.

Will the lease bind the buyer on a sale by the building society?

A. Yes, because the lease has priority over the building society’s mortgage.

B. No, because the lease was granted for a term not exceeding seven years.

C. No, because the lease has not been registered on the register of title to the property.

D. No, because the lease can be overreached.

E. Yes, but only if the tenant consents to the sale.

A

Option A is correct.

The lease is a legal lease and it was created prior to the building society’s mortgage. As a legal lease for a term not exceeding seven years which was in existence when the building society’s mortgage was created, it would be classified as an overriding interest under Schedule 3 para 1 LRA 2002. This is why the lease will have priority over the building society’s mortgage and why the buyer of the property will be bound by the lease.

Option B is wrong. The length of the lease term does not dictate whether the lease will bind a buyer from the building society. As the lease was created prior to the mortgage, the application of the enforceability rules will determine whether the lease has priority.

Option C is wrong. Since a legal lease for a term not exceeding seven years is an overriding interest under Schedule 3 para 1 LRA 2002, it is enforceable even though it does not appear on the register of title.

Option D is wrong because a lease cannot be overreached.

Option E is wrong. The lease has priority over the building society’s mortgage, as indicated above, so the tenant does not have to consent to the sale for the lease to bind the buyer.

85
Q

Three years ago, the owner of registered freehold land granted a lease over the land to a tenant, by deed, for a term of seven years. One year later, the owner created a mortgage by deed over the land in favour of a building society. The mortgage was registered at Land Registry immediately following its creation.

Last year, the owner created another mortgage by deed over the land, this time in favour of a bank. This mortgage was also registered at the Land Registry immediately following its creation. The legal date for redemption has passed in respect of both mortgages. The owner is four months in arrears of interest on the mortgage to the building society.

Which ONE of the following statements best describes the exercise of the power of sale by the building society and the priority between the three interests in the land?

A. The building society can exercise its power of sale and will be able to sell the land free from the lease and the bank’s mortgage.

B. The building society can exercise its power of sale and will be able to sell the land free from the bank’s mortgage but may have to sell subject to the lease.

C. The building society can exercise its power of sale and will be able to sell the land free from the lease, but subject to the bank’s mortgage.

D. The building society can exercise its power of sale but will sell the land subject to both the lease and the bank’s mortgage.

E. The building society cannot exercise its power of sale as the lease was created before its mortgage and a sale will prejudice the tenant.

A

Option B is the correct answer. As the building society’s mortgage was created by deed, the power of sale is implied into the mortgage by s101 LPA 1925. The power of sale has arisen, as the legal date of redemption has passed and the power is exercisable, as there are more than two months arrears of interest (see s103(ii) LPA 1925). Accordingly, the building society can exercise its power of sale.

Priority between mortgages is governed by s48 LRA 2002 and the first to appear on the register takes priority. The building society’s mortgage was registered first and therefore has priority over the bank’s mortgage and the building society can sell free from it.

The lease was created by deed and as it was granted for a term not exceeding seven years, it would be classified as an overriding interest under Schedule 3 para 1 of the LRA 2002. As it was in existence at the date of creation of the building society’s mortgage, it will have priority over it and the building society will have to sell subject to it (see s104 LPA 1925).

Option A is wrong because it suggests that the building society’s mortgage has priority over the lease.

Option C is wrong because it suggests that the bank’s mortgage has priority over the building society’s mortgage.

Option D is wrong because it suggests that both the bank’s mortgage and the lease have priority over the building society’s mortgage.

Option E is wrong because the fact that the lease was created before the building society’s mortgage does not prevent the power of sale being exercised but, if the lease takes priority, the building society will sell subject to the lease, i.e. the buyer from the building society will not get vacant possession.

86
Q

A freehold owner grants a five-year lease to a tenant. The document creating the lease is described as a deed but the freehold owner’s signature is not witnessed and the document does not contain all the terms agreed by the parties. However, the tenant immediately moved into the property and has paid the freehold owner a regular monthly rent at the market rate.

Has a legal lease been created?

A. Yes, as a five-year lease can be created without any formality.

B. Yes, it is a legal periodic tenancy created by parol due to exclusive possession and payment of rent.

C. No, in order to be a legal lease a deed is always required.

D. No, the document is not a deed but equity may recognise the agreement.

E. No, it is a licence as the agreement does not contain all the expressly agreed terms.

A

The correct option is B.

The document creating the lease does not meet the requirements for a deed. However, certain leases do not require a deed. Therefore option C is wrong.

Equity would not intervene as the document does not contain all the expressly agreed terms. Therefore, option D is wrong.

The tenant immediately took possession and paid a monthly rent at market rate with no fine or premium, thus, creating a monthly periodic tenancy. Therefore, option E is wrong and option B is correct.

A fixed term five-year lease cannot be created informally. Therefore, option A is wrong.

87
Q

You are acting on behalf of a client who is considering buying a registered freehold property. The client is concerned that the property may be subject to leases. You have looked at the Land Registry official copy of the title to the property and have found no reference to any leases.

Which one of the following statements best describes whether the property may be subject to leases?

A. There are no legal leases affecting the property.

B. There could be legal leases for 3 years or less affecting the property which are not in writing and are binding although not registered.

C. There could be legal leases for over 7 years affecting the property and these could be binding even though they are not on the register.

D. There could be legal leases for more than 3 years but less than 7 years affecting the property which are not in writing and are binding even though not registered.

E. All legal leases, whether they are in writing or not, are automatically binding if they are completed by the time the buyer registers its purchase of the property.

A

Option B is the best answer.

A legal lease must be created by deed unless it falls within the s.54 parol lease exception (3 years or less, market rent, no fine or premium and right to immediate possession). So, B is the best answer as it correctly describes the parol lease exception and the position of these leases under Schedule 3 para 1.

A legal lease of over 7 years is a registrable disposition (s27 LRA 2002) and must be completed by registration in order to bind a buyer by the date of registration of the buyer’s interest (s 29 LRA 2002). A legal lease of 7 years and under is an overriding interest under Schedule 3 paragraph 1 LRA 2002 if the lease is in existence at the time of completion of the buyer’s purchase of the property.

Therefore, Option A is wrong as there could be a legal lease affecting the property even though it is not on the register.

Option C is also wrong as legal leases of over 7 years should be registered in order to be binding (prior to LRA 2002, the term of years for a lease to be overriding was 21 years or less so this answer could potentially be correct but the question asks for the “best” description).

Option D is wrong as, although legal leases are overriding if 7 years or under, those over 3 years can only be legal if created by deed.

Option E is wrong for two reasons – all legal leases are not automatically binding and the date by which a lease has to be created for the purposes of an overriding interest under Schedule 3 paragraph 1 is the date of completion of the purchase not the day of registration.

88
Q

A lease created seven years ago contained a covenant, imposed on the tenant by the landlord, requiring the tenant to decorate the interior of the property every three years. Both the landlord and tenant have sold their respective interests to successors. The new landlord has discovered that the new tenant has breached this obligation.

Which statement most accurately describes the liability between the new landlord and new tenant?

A. The benefit of the covenant can pass from the original landlord to the new landlord because it has reference to the subject matter of the lease.

B. The new tenant is bound by the covenant as the burden will pass to the new tenant if the covenant is not expressed to be personal.

C. The new tenant has the burden of the lease covenant as it touches and concerns the land.

D. An authorised guarantee agreement will ensure that the new tenant guarantees that the original tenant will perform the lease covenants.

E. The original landlord will have retained the right to pursue the new tenant when the lease reversion was sold.

A

Option B is the correct option because the burden of this covenant will have passed to the new tenant as the covenant was not expressed to be personal.

Option A is wrong because this lease is a “new” lease and therefore the benefit of the covenant will pass as all covenants pass to the successor except those that are expressly stated to be personal.

Option C is wrong because this lease is a “new” lease and therefore the burden of the covenant will pass because all covenants pass to the successors except those that are expressly stated to be personal rather than being determined by an assessment of whether they touch and concern the land or not.

Option D is wrong because an authorised guarantee agreement will ensure that the original tenant guarantees that the new tenant will perform the lease covenants, not that the new tenant guarantees that the original tenant will perform the lease covenants.

Option E is wrong because whether the original landlord retains the benefit of the leasehold covenants will depend upon whether the original landlord was released from the burden of his leasehold covenants by the original tenant or the court.

89
Q

In 2015, a freehold owner, granted a 30-year commercial lease (by deed) to a clothes retailer. In 2017, the lease was assigned to a sweetshop. In 2018, the lease was assigned to a newsagent. In 2019, the lease was assigned to a bookseller. Each assignment was by deed and with the freehold owner’s consent. The freehold owner required the provision of an authorised guarantee agreement as a condition of giving consent on each assignment. The bookseller has failed to pay the latest quarter’s rent.

From whom can the freehold owner recover the outstanding rent?

A. From the newsagent and the bookseller only.

B. From the bookseller only.

C. From the clothes retailer and the bookseller only.

D. From the clothes retailer, the sweetshop, the newsagent and the bookseller.

E. From the newsagent only.

A

The correct option is A.

The lease was created after 1 January 1996 and is, therefore, governed by the LT(C)A 1995. This means that, upon each assignment, the outgoing tenant is released from liability under the lease unless they have provided an AGA (ss 5 and 16 LT(C)A 1995). The benefit and burden of all covenants pass to the assignee (s 3).

An AGA only guarantees the immediate assignee. Upon a further assignment the AGA ceases to have effect (s 16(4) LT(C)A 1995).

The result is that the AGAs given by the clothes retailer and the sweetshop are no longer of any effect and the freehold owner can only pursue the bookseller (the current tenant) and/or the newsagent as a consequence of the AGA.

Options B, C, D and E are, therefore, wrong.

90
Q

A freehold owner grants a lease of a retail unit to a pharmacist in 1994 for 40 years. In 2000 the pharmacist sold the lease to a shopkeeper. In 2010 the shopkeeper sold the lease to a florist. In each case the freehold owner granted consent to the assignment taking place. In 2015 the freehold owner sold the freehold reversion to a developer. It is now 2024 and the florist has not paid the December quarter’s rent to the developer.

Who can the developer sue for the rent?

A. The developer can sue the florist, the shopkeeper and the pharmacist for the rent.

B. The developer can only sue the florist and the pharmacist for the rent.

C. The developer can only sue the florist for the rent.

D. The developer can sue the florist and the pharmacist for the rent and can only sue the shopkeeper for the rent if the shopkeeper gave a direct covenant to the freehold owner when she bought the lease from the pharmacist.

E. The developer cannot sue anyone as she does not have the benefit of the covenant to pay rent.

A

Option D is the best answer.

This is an old lease so the position on privity of contract applies. Option D is the best answer as it establishes that the shopkeeper may only be sued if they gave a direct covenant to the freehold owner.

The developer will have acquired the benefit of the covenants under s41 LPA 1925 so Option E is wrong in that they can sue someone.

The pharmacist will always be liable under privity of contract and the florist will be liable under privity of estate so Options C and E are wrong.

Options A and B are on the face of it correct but only if we know whether or not the shopkeeper gave a direct covenant to the freehold owner to enable privity of contract to subsist.

91
Q

The freeholder of a retail store granted a 15-year lease to a newsagent in 2015. The newsagent recently assigned the lease to a clothing retailer. The freeholder consented to this assignment and, other than checking the clothing retailer’s financial accounts, did not require either the newsagent or the clothing retailer to sign any documentation in addition to the assignment.

The clothing retailer has not paid the rent due on the lease following the assignment. The freeholder is threatening to take action to recover the unpaid rent.

Will the newsagent be liable to pay any of the rent properly owing to the freeholder?

A. Yes, because the newsagent remains liable for all covenants in the lease.

B. Yes, because the newsagent remains liable by virtue of the privity of estate that still exists.

C. Yes, because the newsagent remains liable for the rent during the term of the lease but will be able to seek an indemnity from the clothing retailer.

D. No, because the newsagent was automatically released from any liability on the assignment of the lease.

E. No, because on assignment the newsagent was automatically released from the burden of all covenants that touch and concern the land.

A

Option D is correct.

This is a ‘new lease’ granted after 1 January 1996. In relation to these leases, the original tenant is bound by the covenants of the lease only whilst the lease is vested in them. Upon an assignment, all covenants pass to the incoming tenant and the outgoing tenant is released from any liability under the lease (unless the outgoing tenant has signed an authorised guarantee agreement and the facts indicate that no such agreement has been signed).

Option A is wrong. Under a ‘new lease’, the original tenant is released from its promises under the lease upon assignment.

Option B is wrong. Once a lease has been assigned, there is no privity of estate between a landlord and the assignor. (In any event, the concepts of ‘privity of contract’ and ‘privity of estate’ are best ignored when considering new leases, having been replaced with the statutory regime under the Landlord and Tenant (Covenants) Act 1995.)

Option C is wrong. The newsagent will not need to seek an indemnity against the clothing retailer in this case because the newsagent will not be liable for the unpaid rent in the first place.

Option E is not the best answer. The newsagent will not remain liable for any covenants under the lease, save for those which are expressed to be personal. The covenants do not need to touch and concern the land for the burden to pass to the assignee.

92
Q

Two brothers occupy a flat under a lease granted in 1995 for a term of 99 years. The lease was originally granted to a married couple, who assigned the lease to the brothers in 2010 in accordance with the terms of the lease.

The lease contains a provision stating that the tenant will not make any internal non-structural alterations to the flat without obtaining the landlord’s consent.

The lease also contains a provision stating that the landlord may “re-enter” the flat at any time after any breach of any of the tenant’s covenants in the lease. That provision also goes on to state that if the landlord re-enters the flat, “the lease shall immediately end, but without prejudice to any right or remedy of the landlord in respect of any breach of covenant by the tenant”.

The landlord has discovered that the brothers recently made some internal non-structural alterations without obtaining the landlord’s consent.

What remedy for the brothers’ breach of covenant can the landlord obtain against the married couple?

A. Forfeiture.

B. Injunction.

C. Damages.

D. Specific Performance.

E. Debt action.

A

Option C is correct.

The question is asking about the remedies available against the original tenants under the lease. The lease was granted in 1995 and is therefore an old lease. Under the rules governing enforceability of covenants in old leases, the married couple, as original tenants, will remain liable on the tenant’s covenants for the duration of the lease.

The covenant that the brothers have breached is a non-rent restrictive covenant. The only remedy available for breach of a non-rent covenant against the original tenant under an old lease is damages.

Therefore, all the remaining options are wrong. In addition, option D is wrong because specific performance would only be a remedy potentially available in relation to the breach of a positive covenant.

Option E is also wrong because a debt action is a remedy that is pursued in relation to a breach of the covenant to pay rent.

93
Q

The freehold owner of a shop agreed to give exclusive possession of the shop to a jewellery designer for a period of three years. The agreement, which contained all the terms agreed between the parties, was contained in a document which only the jewellery designer signed.

The jewellery designer paid a premium of £5,000 to the freehold owner and took possession of the shop immediately afterwards. The document provided for the jewellery designer to pay a monthly rent, which represented the market rent for retail properties of that size in that area.

What is the nature of the jewellery designer’s interest in the shop?

A. A legal fixed term lease for three years.

B. A legal periodic lease for a period of one month.

C. An equitable fixed term lease for three years.

D. An easement for a fixed period of three years.

E. A licence for a fixed term of three years.

A

Option E is the correct answer because the formalities and requirements for creating a legal lease and, in the alternative, an equitable lease have not been satisfied on these facts. The occupier therefore has a licence rather than a lease.

Options A and B are wrong. Although a deed is not required to create a legal lease for a fixed term of three years, the requirements of s54(2) LPA 1925 need to be satisfied for such a lease to be legal (this is called the parol lease exception). The lease is for a term not exceeding three years, the lease has taken effect in possession and the rent payable is the best rent reasonably obtainable (i.e. a market rent) but the tenant has paid a premium and the payment of the premium prevents this lease from falling within the parol lease exception.

There is also another reason why Option B is wrong. Had the requirements of s54(2) LPA 1925 been satisfied on these facts, the lease would have been a legal lease for fixed term of three years rather than a legal periodic lease for one month.

Option C is wrong. The lease cannot be an equitable fixed term lease for three years because the document was signed by the jewellery designer only and not by the freehold owner. This is one of the formalities stipulated in s2 LP(MP)A 1989 (the other required formalities have been satisfied: the document is in writing and records the agreed terms).

94
Q

In 1995, the freehold owner granted a commercial lease (by deed) to a company for a term of 40 years. The lease contained a repair obligation on the part of the tenant. The company quickly expanded its business and moved to larger premises and assigned the lease to a distributor in 2010. In 2015, the lease was assigned to a warehouse business. Each assignment was by deed and with the consent of the freehold owner. The property is in disrepair and the warehouse business does not have the financial resources to undertake the work. The freehold owner does not wish to bring the lease to an end as it would be hard to find a new tenant.

Which of the following provides the best advice to the freehold owner?

A. Forfeit the lease and relet the property.

B. Enter the property, conduct the repairs and recover the cost from the warehouse business as a debt due.

C. Pursue a claim for damages for breach of repair against the distributor via privity of estate.

D. Pursue a claim in damages for breach of repair against the warehouse business via privity of estate.

E. Pursue a claim in damages for breach of repair against the company via privity of contract.

A

The correct option is E.

The freehold owner does not wish to end the lease so forfeiture is not a good option. Option A is, therefore, wrong.

The warehouse business does not have the resources to pay for the repairs so it is not sensible to pursue it either via privity of estate or a Jervis v Harris self-help remedy. Options B and D are, therefore, wrong.

The freehold owner can pursue a damages claim against the original tenant via privity of contract. As the business expanded, the original tenant seems a better target to recover the cost of repair.
Option E is, therefore, the best answer.

The distributor was only responsible for the covenant to repair whilst the lease was vested in him. Option C is, therefore, wrong.

95
Q

Is the following statement True or False?

If A grants B the right to occupy an office without payment of rent until such time as it is required for A’s own business, B will have been granted a lease of the property.

A

The statement is False.

There is no certainty to the term. (Note: a tenancy may still arise where there is certainty of duration and exclusive possession, even where no rent is payable Ashburn Anstalt v Arnold [1989]).

96
Q

In the event that the title to the freehold is registered, which one of the following statements is correct?

A. A legal monthly tenancy is an overriding interest under Schedule 3 paragraph 1 LRA 2002.

B. An equitable lease must be registered as an interest affecting a registered estate to be binding on a successor landlord.

C. A tenant of a legal lease of 7 years or less has to satisfy the conditions in Schedule 3 paragraph 2 LRA 2002 for the lease to be an overriding interest.

D. A legal lease of more than 7 years will never require registration as it will be an overriding interest if the tenant is in actual occupation.

A

Only statement A is correct.

Legal leases granted for a term not exceeding 7 years are unregistered interests which override under Schedule 3 paragraph 1 LRA 2002: a legal monthly tenancy would fall within this category. Statement C is therefore wrong as a legal lease of 7 years or less would also come within Schedule 3 paragraph 1 and it would not be necessary for the tenant to satisfy the conditions in Schedule 3 paragraph 2 LRA 2002 for the lease to be an overriding interest.

An equitable lease can bind a successor landlord as an overriding interest if the conditions in Schedule 3 paragraph 2 LRA 2002 are satisfied, therefore statement B is also wrong.

Legal leases of more than 7 years require registration as a registrable disposition and so statement D is wrong as if the lease is not registered, it will not be legal but can only be equitable (note that if it is equitable, it can bind a successor landlord as an overriding interest if the conditions in Schedule 3 paragraph 2 LRA 2002 are satisfied).

97
Q

In 2015 Lucy granted a legal lease of a shop to Troy for a term of 15 years. The lease contained a covenant by the tenant to use the property only as a retail shop. Lucy sold the reversion to Paula in 2021 and two months later Troy sold the lease to Sandra after having entered into an AGA with Paula. Sandra has recently converted the shop into a bar.

Which one of the following statements is correct?

A. Paula can sue both Troy and Sandra.

B. Paula can sue Sandra only.

C. Paula does not have the benefit of the tenant covenant as she is not the original landlord.

D. Paula can sue Troy only.

A

Correct - statement A is correct.

Paula will acquire the benefit of the tenant covenant under s.3(3)(b) of the Landlord and Tenant (Covenants) Act 1995. Sandra will acquire the burden of the covenant under s.3(2)(a) of the Landlord and Tenant (Covenants) Act 1995. Troy is liable on the promise in the AGA given to Paula. Paula can therefore sue both Troy and Sandra.